OST-249 - Medical Coding Prep - Mock Exams
When repeat pathology services are performed on the same day, what modifier code, if any, would be utilized?
-91 (Modifier -91 is utilized in the Pathology section when the exact same procedure (same CPT code) is performed/repeated on the same day.)
Anesthesia services for septoplasty on a healthy 24-year-old male
00160-P1 (Anesthesia code 00160 for procedures on nose and accessory sinuses, and physical status modifier -P1)
Anesthesia services for repair of cleft palate on a normally healthy 6-month-old female
00172-P1, 99100 (Anesthesia code 00172 intraoral procedures, cleft palate and physical status modifier -P1. Also assign 99100 for patients under 1 year (6 months).)
Anesthesia services for multiple laceration repairs of the left leg, 2.5 cm knee, 2.5 cm tibia, 1.5 cm ankle on a 37-year-old normally healthy patient
00400-P1 (Anesthesia code 00400 for integumentary procedures, extremities, and physical status modifier -P1)
Anesthesia services for excision of malignant lesion of the arm, 2.5 cm on a 35-year-old normally healthy female
00400-P1 (Assign code 00400 for integumentary procedures and physical status modifier -P1)
Laceration repairs as follows: 1.5 cm arm, simple; 2.5 cm arm, simple; 1.5 cm arm, intermediate
12031, 12002-51 (Two simple repairs would be summed together; therefore, 1.5 arm and 2.5 arm for a total of 4.0 simple arm = 12002-51. A 1.5 cm intermediate repair of the arm would be assigned 12031. An intermediate repair would be considered more significant and is, therefore, listed first. Subsequent procedures on the skin during the same surgical session will need a modifier -51 appended.) WRONG
Repair, 6.0 cm leg, intermediate
12032 (Assigned intermediate, leg, 2.5-7.0 cm)
Intermediate laceration repair, 2.5 cm, right index finger
12041 (Repair/closure, intermediate, hand/finger, 2.5 cm)
Laceration repairs are as follows: 1.5 cm face, intermediate; 2.5 cm arm, simple; and 1.5 cm arm, intermediate
12051, 12031-51, 12001-51 (Each repair/closure requires a separate code as different anatomical grouping and/or different complexity. Therefore, for intermediate face would be most significant, 12051, followed by intermediate arm, 12031-51, followed by simple arm, 12001-51.)
Chronic wound requiring STSG. Graft was obtained from the left thigh and placed on a 2 x 4 cm chronic wound of the right hand.
15120 (Split-thickness skin graft assigned for recipient site. In this instance, code 15120, 8 sq cm hand, would be appropriate.)
Patient presents with extensive skin cancer of forehead. The site was prepared for skin graft repair with sharp debridement of eschar as well as remodeling of some of the granulation growth site. The STSG was harvested from the right thigh and carefully laid over the defect and trimmed to fit the defect properly. How should this be coded?
15120, 15004 (Two codes are appropriate, one for the preparation of the area (15004) as well as the actual STSG (15120).)
STSG from thigh to arm 4 x 4 cm, STSG from thigh to face 3 x 4 cm
15120, 15100 (Codes for skin grafts are assigned based on recipient site. Therefore, code 15120 is assigned for skin graft to face, and 15100 for skin graft to arm.)
Ulcer at the second MPJ was circumscribed and removed in toto. An Apligraft skin graft was prepared, cut, and sutured into place.
15155 (Tissue cultured graft was applied; therefore, 15155 would be appropriate. No size was specified; therefore, the smallest size would be appropriate. MPJ is the standard medical abbreviation for metacarpophalangeal joint.)
Chronic wound ont the right thigh. FTSG was obtained from the opposite thigh and placed on the 4 x 4 cm wound of the right thigh.
15220 (Graft coded to recipient site, thigh, size is calculated as length x width or 16 sq cm, which codes to 15220.)
A cryosurgery was performed to destroy three premalignant lesions for a patient
17000, 17003 x 2 (Destruction of three premalignant lesions would be assigned 17000 for the first lesion, and 17003 x 2 for the two additional lesions.)
Endoscopic partial left ethmoidectomy, bilateral maxillary antrostomy
31256-50, 31254-51-LT (Bilateral procedures should be assigned before unilateral procedures. In this instance, bilateral maxillary antrostomy was performed endoscopically, 31256-50, and left ethmoidectomy, 31254-51-LT.)
Endoscopic left total ethmoidectomy and partial middle turbinectomy. Sinus endoscope was inserted, and using the sharp edge of the freer elevator, the anterior and posterior ethmoid cells were entered and bone and mucosa removed. Left maxillary sinus ostia were identified and an antrostomy was made. An anterior inferior portion of the left middle turbinate was excised with turbinate scissors.
31256-LT, 31255-51-LT, 30130-51-LT (Left endoscopic maxillary antrostomy is assigned 31256-LT, left ethmoidectomy is assigned 31255-51-LT, and excision of inferior middle turbinate is assigned 30130-51-LT.)
How are three transbronchial biopsies of the lower right and two of the left lobe reported?
31628, 31632 (Transbronchial biopsies are assigned codes based on each lobe; therefore, 31628 should be assigned for the first lobe, and 31632 for the additional lobe.)
A 28-year-old ESRD patient was seen in an outpatient hospital facility for treatment of an obstructed hemodialysis AV graft by percutaneous transluminal balloon angioplasty of the venous portion of the graft. This procedure lasted 45 minutes. The patient had an excellent result and was released to home after recovery from the treatment. Another physician performed the professional radiological supervision and interpretation for this procedure.
36902 (Code 36902 would be assigned for introduction of dialysis circuit with balloon angioplasty. Other code selections are for stent, thrombectomy, and no angiography.)
Ultrasound probe was placed in the rectum and prostatic lengths were determined. Six conventional cryoprobes were placed, eight right and left apical thermal sensors and external sphincter thermal sensors were placed. The warming catheter was advanced into the bladder. Freezing probes 1 and 2 anteriorly continuing posteriorly injecting saline to separate the fascia from the anterior rectal wall. Excellent freezing of the prostate tissue was achieved after which a passive thaw was induced.
55873 (55873 is assigned for cryosurgical ablation of the prostate. Code includes ultrasonic guidance and monitoring, therefore, no additional codes would be assigned.)
This gentleman has prostate cancer and has chosen to have complete transrectal ultrasonography performed for dosimetry purposes. Following calculation of the planned transrectal ultrasound, guidance was provided for percutaneous placement of 1-125 seeds.
55875, 77778-26, 76965-26 (Patient has placement of needles for seeds, assigned code 55875, application of radiation source code 77778-26 and ultrasonic guidance for radiation source application code 76965-26.) WRONG
Abdomen flat/upright, 2 V. Punctate pelvic phleboliths on the left just below the level of the ischial spine. Deformity in left lateral iliac crest, exostoses/bone process versus postbiopsy site.
74019 (Assign 74019 for flat/upright (Decubitus/Erect) views of abdomen. This would be considered 2 views. Found in Diagnostic Radiology, abdomen section.)
CT abdomen with oral contrast
74150 (Would assign 74150 for CT abdomen performed without contrast. Per Radiology guidelines in CPT, oral contrast does not constitute "with contrast.")
Left heart catheterization with left ventriculogram and right coronary vessels to perform PTCA right coronary and left circumflex. Intracoronary stent right coronary x2, left circumflex x1, left descending vessel x1
92928-RC, 92929-LC, 92929-LD (As the left heart catheterization is the "approach" by which the interventional procedures are performed, it would not be codeable. Therefore, 92928 is assigned for right coronary (only one per vessel), 92929-LC for left circumflex, and 92929-LD for left descending.) WRONG
When performing the technical component only for an electrocardiogram, what code(s) should be assigned?
93005 (The specific code for EKG technical portion only is 93005.) WRONG
Electrocardiogram, interpretation and report only
93010 (Code 93010 should be assigned for electrocardiogram when interpretation and report only is performed.)
Cardiac stress testing, complete, including performance of test, supervision, interpretation, and report
93015 (Cardiac stress test should be assigned 93015 for complete service, which, according to CPT guidelines, includes performance of the test, supervision, interpretation, and report.)
Cardiovascular stress test, technical, tracing only
93017 (Assign 93017 for cardiovascular stress test, tracing only from Cardiovascular section.)
External ECG with 48-hour continuous rhythm testing during which analysis was performed. The report was reviewed and interpretation completed for evaluation of change to the pacemaker system. The report conclusion stated predominant rhythm of atrial fibrillation with noncontrolled left ventricular rate.
93224 (ECG monitoring was performed and interpreted; therefore, code 93224 would be assigned.)
Coronary angiography by catheter placement in the coronary artery, supervision, and interpretation
93454 (Coronary angiography by catheter placement in the coronary artery, supervision, and interpretation is assigned code 93454.)
A 16-year-old patient for initial visit, referred by physical therapist at school. Injured foot in track practice and has been complaining of right ankle pain for about a month. Detailed history, detailed exam, and low MDM were performed.
99203 (Detailed Hx, detailed exam, and low MDM would be assigned 99203.)
Squamous cell carcinoma in situ, floor of mouth
)D00.06 (Code as neoplasm, mouth, floor, and utilizing in situ column of neoplasm table
Fraud is defined as
making false statement or misrepresentation of facts on claims.
The most aggressive form of skin cancer is
malignant melanoma. (Malignant melanoma is the most aggressive skin cancer. Basal cell and squamous cell carcinoma are also forms of skin cancer, and the most common, but are not considered the most aggressive. Purpura is a purple-colored rash on the skin that is caused by the internal bleeding of small vessels.)
The connective tissue membranes that protect the brain and spinal cord are known as
meninges (The meninges protect the brain and spinal cord.)
What modifier is appended when multiple procedures are performed during the same operative session, through the same approach, and in the same anatomical site, and when all are appropriate to code/bill?
modifier -51 (Modifier -51 is appended for multiple procedures through the same approach and same anatomical site when appropriate.)
Arthro pertains to which anatomical system?
musculoskeletal (Artho, which means "joint," would refer to the joints, which are part of the musculoskeletal systems. The integumentary system would be skin; digestive would be the stomach and other digestive organs; and the nervous system would involve the nerves, spine, and brain. Therefore, the correct answer would be the musculoskeletal system.)
Would it be appropriate to bill CPT code 29870-RT in conjunction with 29880-RT?
no (No, CPT code 29870 is diagnostic and not billable with a definitive surgical procedure on the same anatomical site.)
The sense of smell is also referred to as
olfaction (Olfaction refers to the sense of smell. Inhalation refers to the drawing of air or other substances into the lungs. Gustation refers to the act of tasting. Sinusitis refers to an inflammatory process that affects the lining of the sinuses/nose.)
Per NCCI guidelines, when multiple lesions are excised through the same incision, how many lesion codes are reportable?
one (Only one code is assigned if more than one lesion is removed through the same incision/excisional site.)
A condition in which bones become thin and result in a decrease in bone density is referred to as
osteoporosis ("Osteo" means "bone," and "porosis" means "abnormal condition of porous"; osteoporosis refers to a condition where the bones become thin and there is a decrease in bone density. While the other answer choices are disorders related to the bone and the joint, they do not refer to decreased bone density; therefore, osteoporosis is the most appropriate.)
The surgical creation of an opening is called
ostomy (All the choices refer to procedures; however, only one, ostomy, refers to creating an artificial opening, such as a colosotomy, where an artificial opening is created between the abdomen and colon when a portion of the colon has been removed, usually for disease.)
The primary internal female reproductive organs are
ovaries (The ovaries are considered the primary internal female reproductive organs.)
What additional criteria for a Medicare claim should be considered in the final assignment of E/M codes that are not contained in the CPT manual?
overarching medically necessary criteria as defined by CMS (The overarching criteria was developed specifically by CMS/Medicare and should be utilized in determining the E/M codes for services for the carriers that utilized CMS/Medicare guidelines. Time criteria is already part of the original E/M guidelines in the CPT manual.)
The period of time before a child/baby is born is termed as
prenatal period (Prenatal (before birth) is the period before the baby is born. Perinatal is the period surrounding pregnancy, while postpartum is the period immediately following birth. Parturition refers to childbirth, commonly known as labor and delivery.)
Tachycardia refers to
rapid heartbeat.
In addition to the usual information submitted on each claim, what additional information must be reported when submitting ancillary services?
referring/requesting physician and NPI number
The RICE protocol is utilized for treating sprains and strains and involves
rest, ice, compression, and elevation.
The diaphysis of a long bone is also referred to as the
shaft
Any pathological change in skin tissue is called a(n)
skin lesion (Any pathological change in skin tissue is referred to as a skin lesion. The other answer choices are specific types of skin lesions or skin disorders.) WRONG
According to CPT guidelines, in addition to the appropriate anesthesia CPT code(s) and modifier code(s), what other anesthesia procedural information is required to correctly report anesthesia services?
start time and stop time (Start time and stop time are required to appropriately assign units of services for anesthesia.)
Contraction of the atria and ventricles is referred to as
systole
Hemostasis refers to
the control of bleeding (Hemostasis refers to the control of ("stasis") blood ("hemo").)
The three parts of the small intestine are
the duodenum, the jejunum, and the ileum. (The small intestine consists of the duodenum, the jejunum, and the ileum. The colon is part of the large intestine, the ischium is part of the pelvis, and the stomach is not considered part of the small intestine.)
The outer layer of the skin is referred to as
the epidermis ("Epi" means "outside, outer," and "dermis" means "skin." Therefore, the word for the outer layer of the skin would be "epidermis.")
New patient presents with neck and back pain worsening over the past year. Pain is worse when she bends, relieved sometimes by ibuprofen. Positive aches and weakness in her muscles, tingling and numbness in arms and hands. She also reports headaches. All other systems reviewed and negative. PMH Hysterectomy, Social, recently divorced. Exam performed of eyes, ENT, respiratory, CV, GI, MS, Neur, and VS, pulse and temperature also performed. She was given prescription and set up for an MRI and physical therapy. What E/M should be assigned for this service?
99203 (Hx - Detailed - 99203 Exam - Detailed - 99203 Medical Decision-Making - Moderate - 99204 All three elements must be met; therefore, 99203 would be assigned.) WRONG
New patient arrives for detailed history and exam and moderate MDM.
99203 (Visit limited by history and exam to 99203.)
Blood pressure check, done by the physician's nurse.
99211 (Service performed by nurse 99211.)
Patient presents for follow-up for her hypertension. She is seen by another physician in the same practice. BP, pulse, and temp are taken. Chest is clear. Cardiac normal sinus rhythm. There are no new problems and her hypertension appears to be stable. MDM is straightforward. What E/M service would be assigned?
99212 (Patient is here for follow-up; therefore, established visits 99212 or 99213 only would be appropriate. Patient presents for hypertension and only the affected problem/area are reviewed/examined, therefore, would be considered problem focused, 99212.)
Office visit, expanded problem-focused Hx and exam, low MDM
99213
Outpatient facility visit, expanded problem-focused history/exam, low MDM
99213 (All three elements equal 99213.)
Office visit for 24-year-old patient with acute bronchitis. Expanded problem-focused history and exam are performed.
99213 (Expanded history and exam qualify for 99213; therefore, the two minimum components needed for a 99213 level are met.)
Office visit for follow-up for stable diabetes mellitus. Patient has multiple medical problems; however, all appear stable at this time. No complaints of diplopia, excessive thirst. She has been symptom-free except for some lower back pain in the past few weeks. Exam completed for back, heart, and lungs. Glucose level normal. Assessment: diabetes type II stable.
99213 (For an established patient, only 99213 and 99214 would be correct. Since documentation does not include a detailed exam of the affected system or a detailed history beyond the affected system, the history and exam would be considered expanded problem focused, and therefore, 99213 would be assigned.)
An established patient presents with diagnosis of allergic sinusitis. The physician performs an expanded problem-focused history, exam, and low MDM. Following the completion of the visit, 12 percutaneous scratch tests were performed to determine the origin of the allergens.
99213-25, 95004 x12 (As history, exam, and MDM were performed and decision was made to perform allergy tests, as a result, both an E/M would be assigned 99213-25 as well as 12 scratch tests were also performed; therefore, 95004 x12 would also be appropriate.)
Patient presented for cystourethroscopy due to lower abdominal pain and possible bladder tumor. Cystoscope was inserted and tumor identified and excised. Surgical path verified the mass was malignant.
C67.9 (Documentation prior to conclusion of encounter indicates signs/symptoms and tumor diagnosed as malignant neoplasm, bladder. Coded as primary as not stated otherwise.) WRONG
Ca brain mets from the lung
C79.31, C34.90 (Primary diagnosis is secondary brain carcinoma, primary carcinoma of lung.)
Metastatic neoplasm of the spine
C79.51, C80.1 (Primary diagnosis coded as secondary spine. Since no primary site is listed, assign C80.1, unspecified site.) WRONG
Migraine headache with aura
G43.109 (Code needs to incorporate "with aura" as well as "migraine." Therefore, codes for headache or without sixth digit would not be appropriate.)
CAD would be coded as
I25.10 (Coronary artery disease is coded without angina unless specified otherwise codes to I25.10.)
Patient presents for annual physical. Has a history of hypertension, diabetes, and seizure disorder, all of which were evaluated and considered under control at this time.
Z00.00, I10, E11.9, G40.909 (Chief reason for visit coded as annual physical without abnormal findings. Abnormal findings only coded if new problem or established problem with increased severity/problem.) WRONG
Cardiomegaly refers to
enlargement of the heart. (All choices refer to conditions of the heart or cardiovascular system. However, cardiomegaly would only refer to enlargement ("megaly") of the heart ("cardio").)
Hepatomegaly refers to
enlargement of the liver. (The suffix "megaly" refers to enlargement. In this scenario, "hepato" refers to the liver; therefore, the correct choice would be "hepatomegaly.")
The study of the causes of disease is known as
etiology (Etiology, "study of" "cause." While other choices also contain the suffix "ology" study of, histology is the study of the cells, psychology is the study of the mind, and endocrinology is the study of the endocrine system.)
What penalties may be imposed by the government for fraudulent activities?
exclusion from the Medicare program (Fraudulent activity may result in exclusion from the Medicare program. Exclusion from the Medicare program would result in the inability to file any claims, electronic or other, or be involved in the Medicare program to any extent.)
Mesothelioma is a type of cancer specifically caused by
exposure to asbestos (Mesothelioma is a specific type of cancer caused by excessive exposure to asbestos.)
What body movement involves a straightening of a body part by increasing the angle of a joint?
extension (Extension is the movement that straightens a body part. Dorsiflexion is movement in an upward direction. Flexion is the act of bending. Abduction is movement of a limb or body part away from the midline of the body.) WRONG
The Rule of Nines refers to the
extent of body surface affected by burns primarily in 9% increments. (Extent of body surface area affected by burns primarily in 9% increments) WRONG
The rapid, quivering, noncoordinated contractions of the atria or ventricles is called
fibrillation (Fibrillation refers to irregular contraction of the heart. None of the other choices involve a contraction of the chambers of the heart; therefore, fibrillation is the correct choice.)
L1 refers to which anatomical part?
first lumbar vertebrae (The levels of the vertebrae are referred to by the section (i.e., lumbar (L), thoracic (T), and cervical (C)) followed by the number of the vertebrae (1, 2, 3, etc.). Therefore, the first vertebrae of the lumbar region would be referred to as L1. The first thoracic vertebrae would be referred to as T1, while the first cervical vertebrae would be labeled as C1. Thereofre, L1 is the first lumbar vertebrae.)
A superficial burn, characterized by pain, redness, and swelling is classified as a
first-degree burn (First-degree is characterized by pain, redness, and swelling; second-degree involves blisters; and third-degree involves full-thickness skin loss.)
When the repair of a fracture also necessitates the application of a cast, what services are reportable?
fracture repair only (The initial application of a cast at the time of fracture repair is included in the global procedure for the fracture. No additional services would be reported.)
The ilium, ischium, and pubis make up what bone?
hip bone
The word part meaning excessive or increased is
hyper (The prefix "hyper" refers to excessive, increased, or above the normal. The other choices are also prefixes, however, intra refers to within, hypo refers to the opposite of excessive or increased, and dys is used to indicate bad, difficult, or painful.)
What is one advantage of implementing a compliance program?
identification of potential problems before identified by third-party carriers (Compliance programs will assist in identifying potential problems before they are identified as the result of a third-party audit.)
The longest portion of the small intestine, and the final region, is the
ileum (The ileum is the longest portion of the small intestine. The duodenum and jejunum are also portions of the small intestine but are not the longest portions. The ileocecal valve refers to the valve that guards the opening between the ileum and the cecum.)
When an intravenous infusion is performed as well as an intravenous injection (IVP) during the same encounter, which service is reported first?
infusion, 96365 (The medication infusion, code 96365, should be assigned first when it is the focus of treatment per CPT guidelines.) WRONG
An intravenous push injection that takes place while the nurse is present the entire time is known as
intravenous push injection (IVP) (An intravenous push injection or IVP is assigned for administration of medication that is pushed into the IV line by the nurse who is usually present for the 3-5-minute injection.)
The suffix indicating inflammation is
itis (Itis is the suffix for conditions that involve inflammation. The other choice that involves a condition is osis which refers to an abnormal condition. The remaining suffixes refer to procedures, centesis being a surgical puncture to aspiration/inject, and ectomy being an excisional procedure.)
Cervical laminoplasty to four vertebral segments
63050 (Assign 63050 for laminoplasty performed at the cervical level located in the Posterior Extradural Laminectomy/Laminotomy, Exploration/Decompression Neural Elements, Excision of Herniated Disks.)
Nerve block, by anesthetic, facial nerve
64402 (Code 64402 for facial nerve block located in the Nervous System section, Surgical Procedures on Extracranial Nerves and Autonomic Nervous System.)
A 45-year-old man presents with severe neck pain. The physician examines the patient and makes the diagnosis of cervical nerve impingement and injects an anesthetic agent into the cervical plexus using three injections.
64413 (Injection into the cervical plexus is assigned code 64413 as the three injections are considered one procedure.) WRONG
Bilateral L5-S1 transforaminal epidural steroid injections
64483-50 (Assign code 64483-50 for transforaminal epidural steroid injections found under injection, anesthetic agent, diagnostic, therapeutic somatic nerves.) WRONG
Paravertebral joint injections were performed at the L1-L2 joint, L2-L3 joint.
64493, 64494 (Paravertebral joint injections are assigned per joint; therefore, 64493 and 64494 would be appropriate for two joints, lumbar.)
Carpal tunnel release. A thenar incision was made, and dissection was carried down through the palmar fascia. Transverse carpal ligament was identified and the ulnar border of the transverse carpal ligament was released. External neurolysis was performed freeing the scarred fascia.
64721 (Assigned to code 64721 for neuroplasty, medial nerve at carpal tunnel in Nervous System section.)
Carpal tunnel release, left
64721-LT (Carpal tunnel performed open is assigned a code from the Nervous System, Neuroplasty Code section. Code 64721-LT would be assigned.) WRONG
Cervical sympathectomy
64802 (Sympathectomy on the cervical spine is assigned 64802.)
Radial keratotomy bilateral for elective vision correction
65771-50 (Radial keratotomy, assigned code 65771-50, as performed bilaterally is found in the Eye, Anterior Segment, Keratoplasty section.)
Extracapsular right cataract extraction on a 72-year-old that requires an iris expansion device. Upon completion of phacoemulsification of the opacified lens, an intraocular lens is inserted.
66982-RT (When extracapsular cataract extraction with lens implant is performed that requires more than the usual services performed, assign code 66982-RT per CPT.)
X-rays taken and interpreted, bilateral standing knees, AP
73565 (CPT code 73565 is assigned for bilateral standing needs, therefore, there is no need to assign modifier -50.)
When a bilateral 2 V tibia/fibula x-ray is performed, how should it be reported?
73590-50 (Bilateral procedures for purposes of the CPC exam should be assigned once with the modifier -50.)
Medicaid, Incontinence pull on protective underwear, youth, disposable
T4534 (Code T4534 only code for youth, disposable underwear)
Laceration repair of the left ring finger
12001 (Coded as smallest size and simple complexity as no additional information. Code 12001 is assigned.)
2 V ankle, right
73600-RT (Would assign 73600-RT for 2 view ankle found in Diagnostic Radiology, lower extremity, ankle section.)
BMP with total calcium
80048 (Basic metabolic profile (BMP) with total calcium is assigned disease-oriented panel code 80048.)
When a sigmoidoscopy is performed, what areas are included in the examination?
Entire rectum, sigmoid colon, portion of descending colon (WRONG)
Encounter due to previous history of alcohol dependence. Continue to monitor no current usage.
F10.21 (Considered "history of" alcohol dependence. ICD-10 directs to alcoholism in remission, F10.21.)
Expressive aphasia, history of CVA
F80.1, Z86.73 (Requires two codes, however, does not state that the expressive aphasia was due to the CVA, therefore, cannot assign "residual effects of CVA." Also Z code appropriate for "history of" CVA, not current condition.) WRONG
Medicare patient with history of adematous polyps presents for periodic screening colonoscopy.
G0105 (Patient with history of colon polyps is considered high risk; therefore, code G0105 would be appropriate.)
Under HIPAA, all patients are entitled to: request copies of their records. all of those listed. review of their records. all records be kept confidential except as specified by the patient.
all of those listed
Ballooning of a weakened portion of an arterial wall refers to a(n)
aneurysm (An aneurysm refers to a weakening in the arterial wall, while an embolus is an object carrier in the bloodstream that lodges a vessel. Varicosities are enlarged tortuous vessels and thromboangiitis obliterans is the inflammation and clotting of arteries and vessels of the hands/feet.)
When pathology services are provided, however, not documented as automated versus manual, what code(s) should be assigned?
automated (Per coding guidelines, when not specified, the least significant service would be assumed. In this case, the automated would be the least significant service for the physician to perform.)
The tailbone is a small triangular-shaped bone consisting of three to five fused vertebrae, also referred to as the
coccyx (The coccyx is also known as the tailbone.)
Modifier -26 indicates
codes in CPT for supervision/interpretation.
An artificial opening through the abdominal wall into the colon is referred to as a(n)
colostomy (An artificial opening is termed as an ostomy. Therefore, an artificial opening into the colon would be a "colo/stomy.")
CHF refers to
congestive heart failure.
The portion of the skull that encloses the brain is called
cranium (The cerebrum and cerebellum are both parts of the brain. The cranium, also referred to as the skull, is the bone that encloses the brain.)
The combining form that means hidden is
crypto (The term crypto refers to hidden, or secret. Rhytido refers to wrinkle, conio refers to dust, and xero refers to dry.)
The excision of foreign material or dead or damaged skin to promote healing is known as
debridement (While all the choices are methods for removing skin or tissue, only one, debridement, involves the excision of disease or damaged skin to promote healing.)
How is a neuroplasty procedure described in the CPT Professional Edition?
decompression or freeing intact nerve from scar tissue, including external neurolysis and/or transposition (CPT descriptor for neuroplasty is defined as "decompression or freeing intact nerve from scar tissue, including external neurolysis and/or transposition.")
Skin ulceration caused by prolonged pressure, usually experienced by someone who is bedridden, is called
decubitus ulcer (The only choice that is a skin ulceration is the decubitus ulcer and is usually due to prolonged pressure. It is often referred to as a "pressure ulcer.")
The inflammation of diverticula in the intestines is known as
diverticulitis (Inflammation ("itis") of the diverticula is known as diverticulitis. Colitis and appendicitis are inflammation of the colon and appendix, respectively, while diverticulosis is the abnormal condition or formation of diverticula in the colon.)
A 7-year-old patient has a v-shaped laceration approximately 2.5 cm in length and a small one about 0.5 cm on the right side of the hand. Lacerations are cleaned and closed with 4-0 Ethilon sutures.
12002 (Both repairs/closures are considered simple complexity (as not stated otherwise). Therefore, the repairs would be summed together (2.5 cm + 0.5 cm = 3.0 cm hand).) WRONG
Laceration to eyelid/eyebrow. Right lower eyelid laceration was sutured using 5-0 Vicryl deep sutures and the eyebrow laceration was sutured utilizing 5-0 Vicryl sutures.
12011 (Stated as deep sutures, however, does not specify repair into deep subcutaneous tissue, therefore, assumed simple. Size also not stated, assumed smallest size, therefore, 12011 would be appropriate.)
Anesthesia services for cardiac radiofrequency ablation, 63-year-old with severe atrial fibrillation
00537-P3 (Anesthesia code 00537 for intrathoracic procedures, radiofrequency ablation, and physical status modifier -P3 as stated as severe systemic disease)
Anesthesia for upper GI endoscopy, 55-year-old normally healthy male patient
00731-P1 (Anesthesia code 00731-P1 for upper abdomen, endoscopy, and provides physical status and age documentation)
Anesthesia services for a normally healthy 75-year-old for emergency laparoscopic cholecystectomy for ruptured gallbladder
00790-P1, 99140 (Anesthesia code 00790 for upper abdomen, intraperitoneal procedures, and physical status modifier -P1. Also addition of 99140 as procedure was performed as emergency.)
Anesthesia services for a 82-year-old with severe Parkinson's and hypertension for liver transplant
00796-P3, 99100 (Anesthesia code 00796 for procedures on upper abdomen, liver transplant, and physical status modifier -P3 for severe disease. Also assign qualifying circumstance code 99100 for patients over 70 (82 years).)
Anesthesia for inguinal hernia repair in normally healthy patient
00830-P1 (Anesthesia code 00830 (lower abdomen hernia), physical status modifier -P1)
A normally healthy 50-year-old for anesthesia for repair of an incarcerated inguinal hernia
00830-P1 (Anesthesia code 00830 for lower abdominal hernia procedures and physical status modifier -P1)
Anesthesia services for radical prostatectomy on a 75-year-old with severe CAD, HTN, COPD
00904-P3, 99100 (Anesthesia code 00904 for radical perineal procedure, that is, prostatectomy. Physical status modifier-P3 for severe CAD, HTN, COPD, and qualifying circumstance code 99100 for patients over 70 (aged 75))
General anesthesia for a 75-year-old male with hypertension and Parkinson's disease for transurethral resection of prostate
00914-P2, 99100 (Anesthesia code 00914 for transurethral procedures, that is, transurethral resection of prostate, physical status modifier-P2 as condition of HTN. Qualifying circumstance 99100 for patients over 70 (age 75))
Anesthesia for radical orchiectomy, inguinal on 45-year-old normally healthy male
00926-P1 (Anesthesia code 00926 for perineum, male genital system, and physical status modifier -P1)
Anesthesia services for total knee arthroplasty, right,on a 65-year-old normally healthy female patient
01402-P1 (Anesthesia code 01402 for knee, arthroplasty, and physical status modifier -P1)
Anesthesia services for arthroscopic carpal tunnel repair, left, normally healthy 26-year-old male
01830-P1 (Anesthesia code 01830 for open/arthroscopic surgical procedures wrist/hand and physical status modifier -P1)
Anesthesia services for vaginal delivery only, patient with gestational diabetes, severe
01960-P3 (Anesthesia code 01960 for vaginal delivery, physical status modifier -P3 for severe gestational diabetes) WRONG
Anesthesia services for emergency C-section on a 29-year-old normally healthy female with eclampsia
01961-P1, 99140 (Anesthesia code 01961 is for Cesarean delivery, and physical status modifier is P1 (normally healthy documented), with qualifying circumstance code 99140 as stated as emergency.)
The rib cage is formed by the sternum and how many pairs of ribs?
12 (There are a total of 12 pairs of ribs in the body.)
How is anesthesia time calculated for coding/billing purposes?
10 or 15 minute increments (Time is calculated in 10 or 15 minute increments depending on carrier.)
Anesthesia time is assigned in the days/units column in increments of
10 or 15 minutes depending on carrier.
Procedure Performed: Resection of 3 cm left posterior neck lesion/mass. A natural skin crease incision was made. Care was taken to resect the entire mass, however, taking care to avoid the underlying cranial nerve. Once the entire mass was resected free, the wound was closed in layers, using 3-0 chromic for the deep subcutaneous and 4-0 Prolene for the skin.
11423, 12041 (Two codes are appropriate, one for the excision of the lesion (not stated as malignant, therefore assumed benign). Per CPT, repair/closure may be assigned when more than simple is required. Therefore, also assign code 12041 for intermediate repair.)
A patient presents with a benign face lesion of approximately 4.0 cm. Excision was carried out in the usual fashion. In order to close the skin defect, STSG 2 x 2 cm was obtained from the thigh area and applied to the defect in the cheek, approximately 3.5 x 4.0 cm.
11444, 15120 (Excision of face lesion, 4.0 cm, assumed to be benign would be assigned 11444. Split-thickness skin graft was 3.5 x 4.0 cm and, therefore, would be assigned 15120.)
Excision of a 2 cm lesion on the lower back with 0.5 cm margins with simple repair. Another lesion was excised from the face, 0.5 cm. Two additional lesions on the arm; both lesions, approximately 1.2 cm, were also excised and closed. Surgical pathology indicated that the lower back and both arms lesions were malignant.
11603, 11602, 11602-59, 11440-51 (Four lesions were excised; therefore, four separate codes must be assigned. Three of the lesions were malignant. Lower back, total 3.0 cm, codes to 11603. The two lesions of the arm both code to 11602; therefore, it is listed twice, with modifier -59 on the subsequent code to indicate distinct from other lesion with same code. The remaining lesion of the face would be assigned 11440-51 as subsequent skin procedures performed during the same surgical session are assigned modifier -51.)
Extensive basal cell carcinoma (BCC) of the left retroauricular area. Wide excision of BCC. Skin graft was obtained from the left thigh and placed at the excision site.
11640, 15120 (Two codes would be appropriate: one for the malignant lesion on the face, code 11640 (no size stated), and the other for split-thickness skin graft (not stated as full thickness), code 15120.)
Excision of malignant lesion, 4.0 cm cheek with 8 sq cm advancement flap
14040 (Excision of lesion included in advancement flap, and, therefore, not separately reportable. Advancement flap, cheek is assigned 14040.)
STSG from thigh to arm, to cover a defect 2 x 4 cm
15100 (Only the graft to the recipient site is assigned code in this instance. Therefore, 15100 only is appropriate in this instance.)
Split-thickness skin graft from thigh to cheek, 2 x 3 cm
15120 (Coded as split thickness, cheek, 2 x 3 = 6 sq cm)
A blepharoplasty will be performed on the eyelids. A lower incision line was marked at approximately 5 mm above the lid margin along the crease. Then using a pinch test with forceps, the amount of skin to be resected was determined and marked. An elliptical incision was performed on the left eyelid and the skin was excised. In a similar fashion, the same procedure was performed on the right eye.
15822-50 (Blepharoplasty on bilateral eyelids is assigned 15822-50.)
Patient has burns noted on both hands, blisters intact on left hand as well as right hand. Silvadene and nonadherent dressing applied and wrapped in kling.
16030 (Burns involving blisters are considered second degree. Assign code 16030 as more than one extremity (two hands) was involved.) WRONG
Treatment of molluscum contagiosum by cryotherapy. Patient was given light sedation and the two lesions were exposed. Curettage was attempted, but the lesions were not easily removed; therefore, cryotherapy was used to treat each of the lesions through two freeze/thaw cycles.
17000, 17003 (Cryotherapy for lesions was the definitive treatment; therefore, 17000 would be appropriate for the first lesion, and 17003 for the second lesion. As 17003 is an add-on code, no modifier -51 would be appropriate.)
Mohs surgery on left arm. Reported routine stains on all slides, mapping, and color coding of specimens. The procedure was accomplished in three stages with a total of seven blocks in the second stage and third stage.
17313, 17314 x 2, 17315 x 2 (Mohs surgery coded as 17313 for first blocks, first stage, 17314 x 2 for two additional stages (total of three), and 17315 x 2 for each additional block (two additional in addition to five already coded).) WRONG
Breast biopsy, percutaneous needle
19100 (Assigned code from Integumentary Section, Excision, biopy, breast, percutaneous needle, code 19100)
Using ultrasound guidance, the physician performed a percutaneous needle core biopsy on a suspicious lump on the patient's right breast.
19100-RT (Assign 19100-RT for biopsy for needle core.)
A left curvilinear breast incision was made and tissue divided down to the firm palpable mass. A silk retraction suture was placed through the area in question and dissection performed to remove the spherical-shaped 5 cm tissue sample.
19101-LT (The procedure performed was an incisional breast biopsy only; therefore, 19101-LT would be appropriate.) WRONG
Right breast was prepped, and an incision was made over the breast mass for intended excision. The patient experienced EKG abnormalities and the physician made a decision to discontinue the procedure and schedule it at a later date.
19120-53-RT (Intended procedure was excision of breast. However, due to the patient's medical condition, the physician made a decision to discontinue the procedure. Therefore, the intended procedure 19120-RT would be reported with modifier -53, 19120-53-RT.)
The left breast was prepped and draped in a sterile fashion. An incision from the 3 o'clock position around to the 9 o'clock position on the areolar border on its inferior aspect was made in the skin and extended to the subcutaneous tissue. The breast mass was excised by sharp dissection. The mass was found to be approximately 1.5-2 cm in maximum dimension.
19120-LT, N63.20 (Right breast mass N63.20 would be assigned as the lesion was not diagnosed as malignant. Also, only the breast mass was excised; therefore, code 19120-LT would be assigned.)
U/S-guided needle localization of right breast lesion. Excisional biopsy of right breast lesion; lesion sent to pathology revealed fibroadenoma of the right breast.
19125-RT (Indicates lesion was excised, no biopsy specimen. Therefore, 19125, excision of breast lesion with preoperative radiological marker, would be assigned.)
Lumpectomy. Incision was made over the palpable breast mass and underlying breast tissue was excised, approximately 6 x 4, constituting a partial mastectomy in the lower right quadrant. This contained the palpable abnormality.
19301 (Stated as lumpectomy; therefore, 19301 would be appropriate.)
Right diagnostic arthroscopy was carried out and revealed the rotator cuff and labrum to be intact. There was extensive bursitis in the subacromial space, and although attempted to maintain the CA ligament in its origin from the distal acromion, it was largely released in the process of performing the acromioplasty and a subacromial decompression.
29826-RT (Per CPT when subacromial decompression is performed without other procedures, assign 29826-RT.) WRONG
Arthroscope was inserted and there were areas of diffuse chondromalacia, Grade III that was treated with chondroplasty. The lateral meniscus was torn on its inner third and partial lateral meniscectomy was performed. There was an unrepairable tear on the posterior horn of the medical meniscus and this was also excised.
29880 (Meniscectomy of both medial and lateral compartments was performed. Chondroplasty is included per CPT.)
A patient suffered a penetrating knife wound to his back. A surgeon performed wound exploration with enlargement of the site, debridement, and removal of gravel from the site. The surgeon decided a laparotomy procedure was not necessary at this time.
20102 (Located in the Wound Exploration category, exploration was performed of the back, code 20102.)
Patient presents for status post pin removal following ORIF bimalleolar ankle fracture 2 years ago. One of the pins has apparently dislodged and can be manipulated right under the skin. The pin is removed and area closed with sutures.
20670 (Assign code for removal of implant. Since implant is under the skin, would be considered superficial, therefore, 20670 would be appropriate.)
Right knee diagnostic arthroscopy, partial medial and lateral meniscectomy, chondroplasty with microfracture, medical femoral condyle, chondromalacia patellar chondroplasty, right knee
29880-RT, 29879-51-RT (Arthroscopic microfracture was performed (29879-RT) as well as medial/lateral meniscectomy (29880-RT). Meniscectomy considered more significant and, therefore, listed first. Diagnostic arthroscopy and chondroplasty included in meniscectomy and not separately reportable.)
Internal derangement of the left knee with operative arthroscopy of the left knee with chondroplasty of the patellofemoral joint and the medial compartment with partial medial meniscectomy of the left knee
29881-LT (Arthroscopic medial meniscectomy performed; therefore, 29881-LT would be appropriate.)
Arthroscopic meniscectomy, right knee
29881-RT (Meniscectomy performed arthroscopically would be assigned 29881-RT because not stated as medial and lateral, and, therefore, assumed to be one compartment only.)
Treatment with manipulation of a nasal septal fracture
21337 (Repair of a nasal septal fracture with manipulation is assigned 21337.)
Mass in the left flank at its widest measuring 4 x 2 cm. Incision was extended into the subcutaneous and fascial layers and the lesion was removed in toto.
21931 (A code from the musculoskeletal section would be utilized for excision of a lesion/tumor that extends into the subcutaenous/fascial layers (musculokeletal area). Therefore, code 21931 would be appropriate in this case.)
After adequate anesthesia was obtained, the patient was turned prone in a kneeling position on the spinal table. A lower midline cervical incision was made and the soft tissues divided down to the spinous processes. The soft tissues were stripped way from the lamina down to the facets and discectomies and laminectomies were then carried out at C3-4, C4-5, and C5-6. Interbody fusions were set up for the lower three levels using structural autogenous bone from the iliac crest. The posterior instrumentation of dual titanium rods with hooks were then cut to the appropriate length and bent to confirm to the normal curve. It was then slid immediately onto the bone screws, and at each level, compression was carried out as each of the two bolts were tightened so that the interbody fusions would be snug and as tight as possible.
22551, 22552 X 2, 22842, 20938 (Assign code 22551 for first level (C3-4), and assign 22552 x 2 for two additional levels (C4-5 and C5-6). In addition, assign 22842 for posterior segmental dual rods and 20938 for structural autograft. As both 22842 and 20938 are add-on procedures, modifier -51 is not necessary in this instance.)
Excision of lipoma, shoulder, extending into the deep subcutaneous tissue
23075 (Code found under Excision, Shoulder, Tumor Soft Tissue, Subcutaneous Tissue, less than 3 cm.)
Left diagnostic shoulder arthroscopy was performed which revealed a large rotator cuff tear. Repair of the rotator cuff tear was attempted arthropically; however, it was necessary to perform a mini-arthrotomy to complete the repair of the tear.
23410-LT (Only definitive procedure performed; therefore, only open rotator cuff repair performed, code 23410-LT.)
Patient suffers from a frozen shoulder. The patient was taken to the OR and the right shoulder was manipulated using first abduction. With this abduction, there were multiple fibrous brands, scarred tissue that was felt to be released. Rotation and abduction was performed and the patient appeared to have full motion of her shoulder.
23700-52-RT (Code found under Shoulder, Manipulation. Assign code 23700-52. Modifier -52 should be appended as no application of fixator device was performed.)
Right lateral tennis elbow release with elbow arthrotomy and exploration. Incision was made over the lateral epicondyle. The interval between the ECU and extensor carpi radially longus was identified and split. The extensor carpi radials brevis was subsequently released from its origin onto the lateral humeral epicondyle.
24357-RT (Release of the elbow tendon was performed (tenotomy). Arthrotomy and exploration is included in definitive procedure. Code 24357-RT would be appropriate.)
Closed reduction, left radial head. Initially, traction was applied to the left forearm through the flexed elbow and manipulation of the radial head was carried out with good reduction.
24655-LT (Code 24655 would be located in section Upper Arm/Elbow, Fracture, Dislocation, Closed Radial Head with Traction and Manipulation. -LT modifier would be assigned because the procedure was performed to the left radial head.)
Incision and drainage, deep abscess, forearm
25028 (Code located under Incision, Forearm, Incision and Drainage, Abscess, Deep. The code is 25028.)
ORIF left distal radius, short arm splinting. Standard volar approach was made and the fracture site was identified. Fracture was reduced and T plate was used to hold the fracture on the volar surface. The hardware was placed and visualized fluoroscopically to make certain in good anatomical position.
25607-LT (Open reduction with internal fixation (ORIF) of distal radius is assigned 25607-LT because number of fragments was not identified; therefore, assumed one. Splinting is included in surgical procedure and not separately reportable.) WRONG
Procedure performed: Repair of right little finger distal interphalangeal fusion nonunion with screw. Incision was made, full-thickness flaps were developed, and nonunion site was indented. It was cleaned with ronguer and Preer elevator; bony surfaces were repaired, and an Acutrak II guide wire was placed across the fusion site. It was then placed with a 28 mm screw and overdrilled with a 28 mm mini screw.
26860-F9 (Coded as arthrodesis, interphalangeal joint (code 26860-F9).)
This is a 22-year-old female who presents today with sacroilitis. On the physical exam, there was pain on palpation of the left sacroiliac joint and imaging confirmation was done for the needle positioning. Then 80 mg of Depo-Medrol and 1 mL of bupivacaine at 0.5% was injected into the left sacroiliac joint with a 22 gauge needle.
27096-LT (Code found under Pelvis/Hip, Introduction/Removal, Injection sacroiliac (SI) joint.)
Patient is having ongoing back and right hip pain. The physician elects to perform a sacroiliac injection at an ambulatory surgery center. After sterile prep, the patient is placed prone and under fluoroscopic guidance; the needle is placed into the SI joint with a mixture of 20 mg of Celestone and Marcaine for pain relief.
27096-RT (Coded to Pelvis/Hip, Introduction/Removal, Injection sacroiliac (SI) joint. Assign code 27096-RT.)
How should you report a deep biopsy of soft tissue of the thigh or knee area?
27324 (Coded to 27324, Femur/Thigh, biopsy, soft tissue, deep.)
A 62-year-old female has a mass growing on her right flank for several years. It has finally gotten significantly larger and is beginning to bother her. She is brought to the operating room for definitive excision. An incision was made directly overlying the mass. The mass was down into the subcutaneous tissue and the surgeon encountered a well-encapsulated lipoma approximately 4 cm. This was excised primarily bluntly with a few attachments divided with electrocautery.
27337-RT (Coded to the musculoskeletal section as incision was done into subcutaneous. Found under Femur/Thigh, Excision, Excision Tumor, Subcutaneous, > 3 cm. Code 27337-RT would be appropriate.)
Arthroscopy, left knee with open lateral retinacular release. Arthroscope cannula was introduced, and the anatomy was examined and appears normal. There was a tight lateral retinaculum noted through range of motion of the knee. A longitudinal skin incision was made, incision carried down to the subcutaneous tissue. Nick was made in the lateral patellar retinaculum and lateral patellar release was accomplished. Arthroscopic portal as well as the lateral release incision was closed.
27425-LT (Attempted arthroscopic lateral release; however, completed open. Therefore, only the definitive procedure (open) is coded. Assign 27425-LT only.)
Right knee manipulation and injection. After suitable anesthesia, the patient's knee was gently manipulated. Knee easily flexed to about 85 degrees. It was gentle manipulated further so it flexed to 130 degrees. Full extension was achieved with manipulation. Knee was injected with 80 mg of Kenalog following completion of the manipulation.
27570-RT (Knee manipulation assigned code from Manipulation, knee, code 27570-RT. Injection for postoperative pain management is included in surgical procedure.)
After general anesthesia, the right knee was flexed about 85 degrees. It was then gently manipulated so it flexed to about 125 degrees. Scar tissue bands were heard popping with gentle manipulation. Full extension was achieved and the knee was then injected with 80 mg of Kenalog and Marcaine.
27570-RT (Manipulation of knee under anesthesia is assigned 27570-RT. Injection for postoperative pain is included in procedure and not separately reportable.) WRONG
Septoplasty. Columella reconstruction with cartilage graft. Utilizing a right hemitransfixion incision, bilateral mucoperichondrial and mucoperiosteal flaps were elevated. Remnants of the cartilaginous septum were removed as well as portions of the bony septum and spurs off the maxillary crest. Tears in the mucoperichondrium were reapproximated as best as possible, and then attention was focused on the columella. Excise tissue at the base was excised as well as scar tissue. Tip support was reconstructed with cartilaginous graft obtained from removed cartilaginous septum remnants.
30630, 30520-51 (Nasal septum perforations were performed, which is coded to 30630. In addition, septoplasty was also performed, for which 30520-51 would be assigned.) WRONG
Hammertoe repair, right foot, second toe
28285-T6 (Hammertoe code is 28285. As this can be performed to all toes, the T modifier would be appropriate. In this instance, T6 is to indicate the second toe, right foot.)
Austin bunionectomy. Hallux abducto valvgus of right foot treated with Austin bunionectomy and metatarsal osteotomy of the right foot.
28296-RT (Bunion was excised and distal metatarsal osteotomy performed, described in 28296-RT.)
Application of knee immobilizer, left
29505-LT (No fracture repair; only application of long leg splint. Code 29505-LT.)
Patient tripped and hit right foot on chair during the evening. Radiological exam found he had fractured his great toe and second toe. Physician strapped the toes and the patient was discharged.
29550-RT (Strapping only performed; therefore, 29550-RT assigned only.)
Arthroscopic TMJ repair
29804 (Code 29804, located under Arthroscopy, temporomandibular joint.)
Procedure performed: Bilateral nasal endoscopy with cautery. Complete bilateral nasal endoscopy performed with zero-degree endoscope. Only potential bleeding sites on completed endoscopy were located in the anterior septum on each side. These were cauterized under direct visualization.
31238-50 (Endoscopic repair of nasal hemorrhage was performed bilaterally; therefore, code 31238 with modifier -50 would be appended.) WRONG
Utilizing a 5" sinuscope and microdebrider, the right concha bullosa was resected in its lateral portion. Subsequently, the anterior wall of both ethmoids was removed with the microdebrider, and the hypertrophic mucosa and anterior ethmoid cells were cleared with the microdebrider. The maxillary sinus ostia were cleared of hypertrophic mucosa on the right side followed by the same procedures on the left.
31267-50, 31254-50-51 (Bilateral endoscopic maxillary antrostomies were performed (31267-50) as well as bilateral ethmoidectomies (31254-50), which would also need modifier -51 appended for multiple surgical procedures.)
Anterior laryngoscope was entered and passed to the level of the vocal cords. Polyps were identified on the left false and right true vocal cords. CO2 laser was used to excise the polyps, identified as papillomas.
31540 (Laryngoscopic excision of lesions was performed, which is coded to 31540.)
The old tracheostomy site was palpated for signs of a tracheal defect; none was noted. A vertical incision was made through the old scar until the trachea was reached. There were significant calcifications of the tracheal rings, making identification of the individual rings difficult. A number 11 blade was used to create a vertical incision, and scar tissue over the previous tracheostoma was excised until the endotracheal tube could be visualized. A number 8 nonfenestrated, cuffed tracheostomy tube was easily inserted into the tracheostomy.
31600 (Replaced tracheostomy tube. Not stated as emergency and, therefore, assumed planned, so code 31600 is assigned.)
Bronchoscopy with washings
31622
Bronchoscope inserted through right nostril. Nasopharyngeal mucosa normal; vocal cords and trachea normal. There were moderate, thick mucoid secretions from the trachea that was suctioned. Right main bronchus appeared normal. Washings from the right lower lobe were obtained and sent to microbiology.
31622 (Bronchoscopy was performed with washings, which is included in the dx bronchoscopy, 31622.)
Bronchoscopy, diagnostic
31622 (When only diagnostic bronchoscopy is performed, code 31622 is assigned.)
Bronchoscopy performed through mouth block. Bilateral BAL performed and transbronchial biopsies taken from the right middle and right lower lobes.
31624-50, 31632-51-RT, 31628-51-RT (Bronchoscopic alveolar lavage was performed bilaterally, 31624-50, and transbronchial biopsies of two lobes were taken, codes 31628 and 31632 assigned. Modifier -51 is assigned for all subsequent procedures, and codes are placed in the order of significance; therefore, 31624-50 listed first (as bilateral) followed by 31632-51-RT and 31628-51-RT.)
Patient presented for diagnostic bronchoscopy for possible lung nodule on radiological study. The scope was advanced into the right middle and lower lobe, where a biopsy was obtained of a suspicious nodule.
31625 (Bronchoscopy was performed with biopsy, which codes to 31625. Code 31628 is for transbronchial biopsies, which were not performed in this instance.)
Procedure: The patient was brought to the endoscopy suite and the mouth and throat were anesthetized. The bronchoscope was inserted and advanced through the larynx to the bronchus. The left side was examined first and no abnormalities were appreciated. The bronchoscope was then introduced into the right bronchus. Using fluoroscopic guidance, the tip of the bronchoscope was maneuvered into the area of the mass. Closed biopsy forceps were passed through the channel in the bronchoscope and transbronchial into the right lower lobe. A tissue sample was obtained. There were no other abnormalities appreciated in the right side, and the bronchoscope was removed.
31628 (Transbronchial biopsies performed via a bronchoscope are assigned based on the number of lobes. In this instance, only one lobe was biopsied; therefore, 31628 would be appropriate.)
Bronchoscopy performed with transbronchial biopsies—two biopsies to the right upper lobe and two biopsies to the left lower lobe.
31632, 31628 (Transbronchial biopsies are assigned codes based on each lobe; therefore, 31628 should be assigned for the first lobe and 31632 for the additional lobe.) WRONG
CT-guided lung biopsy. An appropriate area was cleansed and an 18-gauge spring-loaded needle was inserted into a 6 × 5 cm mass in the right base. Three cores of tissue were removed into the biopsy needle.
32405 (When a lung biopsy is performed via needle, code 32405 would be assigned.)
A CT scan identified moderate-sized right pleural effusion in a 60-year-old female. This was estimated to be 800 cc in size and had an appearance of fluid on the CT scan. A surgical puncture using an aspirating needle punctured through the chest tissues utilizing CT imaging and entered the pleural cavity. Fluid was aspirated, draining the effusion under ultrasound guidance using 1% lidocaine as local anesthetic.
32555 (Thoracentesis was performed with imaging; however, no indwelling catheter was placed. Therefore, code 32555 with imaging would be appropriate.)
Thoracentesis with indwelling tube placement
32556 (Thoracentesis was performed with no imaging; however, indwelling catheter was placed, which codes to 32556.)
2 V left foot X-ray for injury. Mildly displaced fracture at the base of the first metatarsal involving the metaphysis. Otherwise negative.
73620-LT (Assign 73620-LT for 2 view left foot X-ray found in Diagnostic Radiology, lower extremity, foot section.)
Thyroid uptake imaging, interpretation only
78012-26 (Thyroid uptake imaging is assigned nuclear medicine code 78012 with modifier -26 for interpretation only.)
Patient had a temporary ventricular pacemaker placed at the start of a procedure. This temporary system was used as support during the procedure only. How would you report the temporary system?
33210 (Insertion of a temporary pacemaker is assigned 33210.)
Placement of temporary cardiac pacemaker, single chamber
33210 (Temporary pacemaker is assigned code 33210.)
Removal of pacemaker pulse generator
33233 (As there is removal only of pulse generator, therefore, 33233 would be appropriate.)
CABG, venous grafting, three vessels
33512 (CABG (coronary artery bypass graft) was performed with venous grafting, three vessels. Therefore, 33512 will be assigned.)
A 67-year-old patient with known CAD and a history of MI was admitted for CABG. A portion of the right radial artery was procured for bypass grafting, and then proceeded to perform two arterial grafts. Then a portion of femoropopliteal vein for graft was harvested.
33534, 35572 (Assign one code for the artery grafting (code 33534), as well as a code for harvesting the femoropopliteal vein for graft (code 35572).) WRONG
Blood transfusion
36430 (Only one transfusion code is assigned, 36430, found in Arteries/Veins, Venous Procedures. The number of units of blood product is assigned HCPCS "P" codes by the facility providing the blood product.)
At the patient's bedside in the hospital, a PICC line is inserted. Using xylocaine local anesthesia, aseptic technique, and ultrasound guidance, a 21-gauge needle was used to aspirate the right cephalic vein of a 72-year-old patient. When blood was obtained, a 0.018-inch platinum tip guide wire was advanced to the central venous circulation. A 6 French dual lumen PICC was introduced through a 6 French peel-away sheath to the SVA RA junction, and after removal of the sheath, the catheter was attached to the skin with a STATLOCK device and flushed with 500 units of heparin in each lumen.
36569 (When a central venous catheter is inserted peripherally, code 36569 is assigned. PICC stands for peripherally inserted central venous catheter.) WRONG
A 52-year-old female has recurrent lymphoma in the axilla. Ultrasound was used to localize the lymph node in question for needle guidance. An 11 blade scalpel was used to perform a small dermatotomy. An 18 × 10 cm BioPince needle was advanced through the dermatotomy to the periphery of the lymph node. A total of four biopsy specimens were obtained. Two specimens were placed in RPMI and two were placed in formalin and sent to laboratory.
38505 (When lymph nodes are biopsied/excised by needle, code 38505 is assigned.)
Tonsillectomy/adenoidectomy, age 20
42821 (Both tonsils and adenoids were excised; therefore, code 42821, for patients over 12, would be assigned.)
A 36-year-old presents for tonsillectomy. Tonsils were grasped with Allis forceps, and tonsil was bluntly dissected free. Identical procedure was performed on the other tonsil as well. The nasopharynx was viewed, and considerable amount of adenoidal tissue was also removed.
42821 (Tonsils and adenoids were excised; therefore, code 42821 would be appropriate for patients over 12.)
Tonsillectomy age 8
42825 (Tonsillectomy only was performed; therefore, 42825 would be appropriate for patients under 12.)
A 13-year-old presents for removal of tonsils. Tonsils are grasped and removed.
42826 (Tonsils only were excised; therefore, code 42826 would be appropriate for patients over 12.)
A 15-year-old male is to have a tonsillectomy performed for chronic tonsillitis and hypertrophied tonsils. A McIvor mouth gag was put in place, and the tongue was depressed. The nasopharynx was digitalized. No significant adenoid tissue was felt. The tonsils were then removed bilaterally by dissection. The uvula was a huge size because of edema, a part of this was removed and the raw surface oversewn with 3-0 chromic catgut.
42826, 42106-51 (Tonsillectomy performed bilaterally (no bilateral modifier is required per CPT). In addition, lesion/area of uvula was excised and assigned code 42106-51.)
Gastroscope was introduced through the mouth and passed into the stomach, through the pylorus and into the second portion of the duodenum. Just inferior to the Z-line, a small polyp was identified and cold biopsied. Rectal exam was performed, and colonoscope was advanced to the cecum. Multiple sessile polyps were removed utilizing snare and cauterized. There were two other tiny polyps that were fulgurated due to their size.
45388, 45385-51, 43239 (Both colonoscopy and EGD (esophagogastroduodenoscopy) were performed. The colonoscopy involved fulgurating polyps (45388) and polypectomy by snare (45385-51). The cauterization of the snared polypectomy is not separately reportable to control bleeding created by the excision. An EGD was also performed with biopsy, which would be assigned 43239.) WRONG
Examination under anesthesia with left lateral internal sphincterotomy. Examination of the anal canal demonstrates a posterior anal fissure. Left lateral mucosa over the internal sphincter was incised and hemostasis achieved. Under direct vision, the sphincter was divided. The mucous membrane was then oversewn in a running locking fashion.
46080 (Code 46080 sphincterotomy includes the division of the sphincter.)
Esophagoscopy with FB removal
43215 (When only esophagoscopy is intended and foreign body is removed, assign 43215.)
Endoscopic ultrasound for esophageal cyst. Video radial echoendoscope was introduced through the esophagus, stomach, and duodenum. Moderate gastritis was visualized. Rest of exam was unremarkable. Ultrasound images were taken showing two cystic lesions in the esophagus.
43237 (Endoscopic ultrasound of the esophagus, stomach, and duodenum is assigned 43237.)
EGD. Scope was passed and visually guided into the esophagus, the stomach, and advanced all the way to the third part of the duodenum. A couple of biopsies were taken from the small bowel mucosa. An esophageal ulcer was examined and biopsy taken as well.
43239 (EGD (esophagogastroduodenoscopy) was performed with biopsies. Code 43239 indicates biopsy, single or multiple, and is only coded once regardless of the number of specimens/locations.)
EGD with biopsy of gastric polyp. Gastroscope was advanced into the duodenum. Biopsies of the antrum are taken, and a small polyp in the stomach is removed with biopsy forceps.
43239 (Multiple biopsies were obtained during the course of the EGD, however, only assigned once regardless of the number of polyps/locations to 43239. When polyps are removed by biopsy forceps, CPT assistant directs to code an EKG with biopsy as well.)
Removal of foreign body from esophagus by scope through guidewire following EGD.
43248 (EGD (esophagogastroduodenoscopy) was performed at which time removal of foreign body through guidewire is assigned 43248.)
A patient diagnosed with GERD presents to the same day surgery department for an upper GI endoscopy. The procedure is done in order to treat the GERD by delivering thermal energy to the muscle of the gastric cardia and lower esophageal sphincter. Anesthesia was administered and as the physician begins the procedure, the patient's blood pressure drops to a dangerously low level. The physician decides not to finish the procedure due to the risk it may cause to the patient.
43257-53 (EGD was performed with thermal energy to muscle of lower esophageal sphincter or gastric cardia for treatment of GERD, and assigned code 43257. Treatment was not completed, when the patient's condition caused the physician to terminate the procedure; therefore, modifier-53 should be appended.)
ERCP with removal of obstructed biliary stent, balloon of CBD stones and sludge, placement of biliary stent. Duodenoscopy was advanced through the esophagus, stomach, and pylorus into the duodenum. An obstructed biliary stent was snared and removed. Selective cannulation of the CBD was performed. Balloon of the CBD was performed, removing soft stones and sludge followed by a new 10 French biliary stent placed across the strictures.
43276, 43264-51 (Exchange of stent was performed, 43276, as well as removal of stones and sludge, 43264. Therefore, code 43276 is assigned, followed by 43264-51.)
C7-C8 were identified and 0.25% bupivacaine was injected as a test dose. Forty mg of Depo-Medrol was injected into the epidural space and needle removed.
62320 (When cervical epidural injection is performed, assign 62320.)
Percutaneous liver biopsy. The upper right abdomen was prepped and 18 gauge coaxial needle was inserted into the right lobe of the liver. Needle tip was placed within one of the lesions and core biopsies X 3 were obtained.
47000 (When percutaneous liver biopsies are performed by needle, code 47000 is assigned. This code is assigned only once regardless of the number of biopsies/specimens obtained.) WRONG
Liver biopsy due to elevated liver enzymes. The area of biopsy site was chosen and a small nick was made on the skin and advanced to the liver capsule. Patient was told to hold her breath, biopsy gun was inserted into the liver, and the obtained biopsy was sent to pathology.
47399 (Biopsies were not obtained by needle or wedge, and, since there are no other codes for liver biopsies, would assign unlisted liver, 47399.) WRONG
Digital rectal exam was performed, and no masses were palpable. Scope was introduced to the cecum. The scope was withdrawn, and there were several areas of liquid stools at various stages. In the sigmoid region, there were two polyps adjacent to each other, both less than 5 mm which were hot biopsied and sent to pathology.
45380 (Colonoscopy was performed with multiple biopsies, therefore 45380 would be appropriate. The hot biopsy polypectomy code (45384) is only appropriate if the polyp, lesion, tumor is excised.) WRONG
When multiple biopsies are performed at four different sites during the course of a colonoscopy, what code(s) should be reported?
45380 once only (CPT code 45380 indicates single or multiple, therefore, it should be assigned only once regardless of the number of biopsies.)
Screening colonoscopy. Colonoscope was inserted and passed through all flexures to the cecum. The patient had a few diverticuloses in the sigmoid colon, and a small sessile polyp in the distal sigmoid was removed by snare.
45385 (Screening colonoscopy was performed. However, a polyp was removed by snare polypectomy. The screening colonoscopy is included, and only 45385 is assigned.)
Colonoscope was introduced and advanced to the cecum. Beyond the cecal valve, there was an area with a cauliflower-like appearance. Several biopsies were taken, hot and cold, of the area in question. In the area of the sigmoid colon, an additional polyp was snared in toto.
45385, 45380-59 (Colonoscopy was performed with polypectomy by snare (45385) and biopsy(ies) of other polyps, 45380 with modifier-59 appended to indicate biopsy(ies) of separate sites.)
A patient was referred by primary care to GI for rectal bleeding. The colonoscope was advanced into the cecum. The patient had a polyp removed by snare, another polyp was hot biopsied, and a separate area of questionable inflammation was also biopsied. The base of the polyps was cauterized to control bleeding.
45385, 45380-59 (One technique is allowed per polyp or lesion; therefore, a code for the polyp removed by snare (45385) would be appropriate as well as a separate code assignment for the biopsy(ies) performed to other sites. The polyp that was hot biopsied was not removed and, therefore, would be included in the 45380 with biopsy code. Cauterization to control bleeding created by the excisions or biopsies is included in the procedures assigned.) WRONG
Laparoscopy cholecystectomy
47562 (Cholecystectomy (excision of gallbladder) was performed laparoscopically; therefore, code 47562 is appropriate.)
A patient is admitted to the hospital for chronic cholecystitis for which a laparoscopic cholecystectomy will be performed. A transverse infraumbilical incision was made sharply dissecting to the subcutaneous tissue down to the fascia using access under direct vision with a Vesi-Port, and a scope was placed into the abdomen. Three other ports were inserted under direct vision. The fundus of the gallbladder was grasped through the lateral port, where multiple adhesions to the gallbladder were taken down sharply and bluntly: The gallbladder appeared chronically inflamed. Dissection was carried out to the right of this, identifying a small cystic duct and artery, was clipped twice proximally, once distally and transected. The gallbladder was then taken down from the bed using electrocautery, delivering it into an endobag and removed from the abdominal cavity with the umbilical port.
47562 (When gallbladder is excised laparoscopically, assign code 47562.)
Cholecystectomy, open
47600 (When gallbladder is excised (cholecystectomy) through open technique, assign 47600.)
Patient scheduled for possible TAH/BSO, however, surgical abdomen is explored and no additional procedures performed.
49000 (Since patient was scheduled for "possible" TAH/BSO following findings from exploratory laparotomy, only exploratory laparotomy would be assigned, code 49000.) WRONG
Laparoscopy, lysis of adhesions and right ovarian cystectomy. A subumbilical incision was made and cannula and ports were inserted. Adhesions from the omentum to the anterior abdominal wall were taken down, and, after freeing up the adhesions, the right ovary was noted to have two ovarian cysts, which were aspirated.
49322 (Code 49322 assigned for laparoscopic aspiration of ovarian cyst(s), found in abdomen, laparoscopy section.)
Inguinal hernia repair, age 4
49500 (Inguinal hernia repair was performed open. As not stated as incarcerated/strangulated, would assign 49500 for patients aged 4.)
An otherwise healthy 22-year-old patient was scheduled for repair of an incarcerated bilateral recurrent inguinal hernia. The surgeon created the incision and started the procedure. At this point, the patient went into shock due to the surgery and the procedure was halted. The patient was stabilized and returned to the recovery room.
49521-50-53 (Repair of a recurrent, incarcerated bilateral inguinal hernia was attempted, however, not completed due to the patient's condition; therefore, code 49521 would need modifier -53 appended, as well as modifier -51 as procedure was attempted bilaterally.)
Hernioplasty to repair a recurrent ventral incarcerated hernia with implantation of mesh for closure. The surgeon completed debridement for necrotizing soft tissue due to infection at another site.
49566, 11005-59, 49568 (Recurrent ventral hernia is repaired through open technique, code 49566 which also requires implantation of mesh, which is separately reported with code 49568. In addition, debridement for necrotizing soft tissue due to infection was performed to another site, code 11005-59.)
A semicircular incision was made in the margin of the umbilicus of this 6-year-old. Dissection was carried down to the fascia. Posterior part of the umbilical skin was detached, and exam showed pinpoint opening in which omentum was stuck. Opening was enlarged, and the omentum was adhesed to the umbilical opening. It was released and returned to the abdominal cavity.
49587 (This represented an incarcerated hernia as the omentum was "stuck" and had to be cut and released and returned to the abdominal cavity. Therefore, code 49587 for an incarcerated hernia on a patient over 5 should be assigned.) WRONG
A 36-year-old female presents for laparoscopic bilateral inguinal hernia repair with open umbilical hernia repair also was performed.
49650-50, 49585 (Bilateral laparoscopic inguinal hernia on a 36 year old, codes to 49650-50. An open umbilical hernia was also repaired, which codes to 49585.) WRONG
Laparoscopic ventral herniorrhaphy with patch. A small incision was made, and the trocar was introduced. In the left lower quadrant, two trocars were placed, and the hernia was easily visualized. The herniated area was removed, and the area was covered with an oval mesh patch measuring approximately 8 X 10. Trocars were removed, and the excess CO2 was allowed to escape, and the skin was closed.
49659 (When trocars are introduced, the procedure is intended to be performed laparoscopically. In this scenario, a ventral hernia repair (orrhaphy) was performed. Therefore, codes from the laparoscopic section of the hernia repairs (GI section) should be assigned. In this case, 49659 would be appropriate.) WRONG
Left renal stone was seen, which was right next to the stent placed previously. Stone was targeted in both AP and lateral views. Starting at an energy of 2 Kv and going to 9 Kv, the stone was fragmented utilizing ESWL. Additional 500 shocks were required for total pulverization of the stone.
50590-LT (Extracorpeal shock wave lithotripsy or ESWL was performed. Code as 50590-LT.)
A 10-day-old male baby is in the OR for a repeat circumcision due to redundant foreskin that caused circumferential irritation from the original circumcision. Anesthetic was injected and an incision was made at base of the foreskin. Foreskin was pulled back and the excess foreskin was taken off and the two raw skin surfaces were sutured together to create a circumferential anastomosis.
54163 (Repeat circumcision is performed, therefore, assign code 54163 from penis, excision section.)
Placement of suprapubic catheter. Approached the site suprapublically through the skin and fascia until the bladder was accessed. Then a stab incision was made, the bladder was aspirated, and a suprapubic catheter was placed which was sewn into place.
51102 (Code 51102 would be assigned for aspirating the bladder and then placing a suprapubic catheter.)
Diagnostic cystourethroscopy is performed, which reveals 2 cm ureter stone.
52000 (Only diagnostic cystourethroscopy was performed, therefore, 52000 would be assigned.)
Diagnostic cystourethroscopy
52000 (When cystourethroscopy only is performed, assign diagnostic cystoscope code 52000.)
Excision of a local lesion of the epididymis.
54830 (Assign code 54830 found in the male genital section, epididymis, excision section.)
Vasectomy
55250 (Excision of the vas deferens, or vasectomy is assigned code 55250.) WRONG
Cystoscopy was introduced into the bladder. Tumor measuring 2 cm was located on the bladder. Using cutting current, this was resected and particles removed through the evacuator. Tumor measuring 1 cm was located on the bladder base, which was also resected along with a third tumor, 2 cm, located on the upper portion of the bladder.
52235 (CPT coding guidelines per CPT Assistant indicate the largest lesion only should be assigned, therefore, 52235 should be assigned for the 2.0 cm lesion removed cystourethroscopically.) WRONG
A patient is diagnosed with a bladder tumor. The physician performs a cystourethroscopy with fulguration and resects a 7 cm bladder tumor.
52240 (Cystourethroscopic resection of 7 cm bladder tumor is assigned code 52240.)
Cystourethroscopy with placement of urethral stent
52282 (When cystoscopy is performed and urethral stent is placed, assign 52282 (note this is urethral NOT ureteral).)
Cystoscopy with ureteroscopy with balloon treatment of ureteral stricture
52341 (Cystoscopic with treatment of ureteral stricture by balloon dilation is assigned 52341.)
Electrosurgical transurethral resection of prostate
52601 (Electrosurgical TUR of the prostate is located in the urinary section, vesical neck and prostate, and assigned code 52601 for electrosurgical resection of prostate.)
A 38-year-old patient with BPH has his prostate removed via a laser enucleation. During this procedure he also has a vasectomy.
52649 (Laser enucleation of the prostate is assigned code 52649 found in the urinary, vesical neck and prostate section.)
Urethral biopsy
53200 (Code 53200 for urethral biopsy located under urinary, urethra section under excision, biopsy.)
A newborn baby underwent a procedure to slit the prepuce to relieve constriction that prevented retraction of the foreskin over the head of the penis. The slit tissue was sutured at the divided skin to control bleeding.
54000 (Procedure to slit the prepuce of the penis is assigned 54000 found in the male genital section, Penis, incision section.)
Circumcision of 1 day old infant. Base of the penis was cleansed. Incision was made down to the apex. The foreskin was then retracted and the foreskin attachment was cross-clamped and removed with scissors.
54150 (Circumcision performed utilizing clamp, assign 54150.) WRONG
Abdomen, vulva, and vagina were prepped and the cervix was visualized extending outside the vagina. Anterior incision was made 2 cm from cervix. The denuded strip was approximately 2-3 cm in width. The denuded strip was grasped with the clamp, and posterior incision made. Procedure was completed by suturing the anterior vaginal mucosa to the posterior vaginal mucosa and then successive suturing to elevate the uterus and anterior bladder wall.
57120 (The suturing of the vagina is performed with strips to correct prolapsed vagina. Assign code 57120.)
Cytoscopy with urethral dilation with a posterior repair of rectocele. 21 French panendoscope was inserted and the urethra was dilated from 16 to 38 French female sound. The midline of the rectocele was marked and a small triangular tissue of skin was removed. Posterior vaginal wall was then dissected lateral to the lateral pelvic well. At the apex of the vagina, a rectocele was identified and repaired. Posterior vaginal wall was closed at which time extra skin was also removed.
57250 (Posterior colporrhapy was performed and repair of rectocele accomplished. Assign 57250.) WRONG
Dilation and curettage
58120 (Dilation and curettage of the corpus uteri (uterus) is assigned code 58120.)
Diagnostic amniocentesis
59000 (Diagnostic amniocentesis is found in the maternity services section of the female genital surgery section and assigned code 59000.)
Basic metabolic panel with ionized calcium
80047 (Code 80047 is assigned for basic metabolic profile ionized calcium.)
Evaluation under anesthesia, hysteroscopy, dilation and curettage, and endometrial ablation. Using a Storz hysteroscope, a hysteroscopy was performed, which revealed uterus consistent with menorrhagia. Then, using an Ethicon endometrial ablation apparatus, endometrial ablation of the cavity for 8 minutes at 87 degrees Celsius.
58563 (Hysteroscopic ablation was performed, which is assigned code 58563. Code found in laparoscopy/hysteroscopic procedures, corpus uteri section.)
The surgeon finds an enlarged ovarian cyst on the left and an attempt is made to remove the cyst. However, multiple attempts failed to remove the cyst, and the surgeon proceeded with a left salpingo-oophorectomy.
58720-LT (Only the definitive procedure is reported, therefore, salpingo-oophorectomy is reported with code 58720-LT.) WRONG
Vaginal delivery of normal healthy baby girl.
59409 (Code 59409 is assigned for vaginal delivery only. Scenario does not mention that antepartum and/or postpartum care was performed by the same physician; therefore, the vaginal delivery code only would be appropriate.)
Vaginal delivery only
59409 (When only vaginal delivery is performed, assign 59409 for delivery only code.)
Cervix was dilated and a #10 suction cannula introduced. The anterior, posterior, and lateral walls of the endometrial cavity were curetted, yielding moderate amounts of retained products of conception, which were removed.
59812 (An incomplete abortion is a spontaneous abortion with retained products of conception. Therefore, this procedure would be coded as 59812.)
Treatment of incomplete abortion.
59812 (An incomplete abortion is a spontaneous abortion with retained products of conception. Therefore, this procedure would be coded as 59812.)
Open procedure replacement of cranial nerve neurostimulator. The pulse generator was connected to the neurostimulator array inserted previously, tested, and repositioned to ensure maximum effectiveness. The pulse generator was placed and sutured into a created subcutaneous pocket. Again, the system is tested to ensure proper functionality.
61885 (Assign 61885 only for pulse generator replacement of the neurostimulator. Array was previously placed and, therefore, not separately reportable for this surgical procedure.) WRONG
Right frontal ventriculoperitoneal shunt placement with Codman programmable valve
62223 (Placement of a ventriculoperitoneal shunt is assigned 62223.) WRONG
A patient with uncontrolled low back pain presents and has implantation of a nonprogrammable intrathecal morphine pump.
62361 (Implantation of a nonprogrammable intrathecal pump is assigned 62361 for nonprogrammable pump located in the Spine and Spinal Cord section of the Nervous System.)
A craniectomy for Chiari malformation. Once the posterior inferior scalp was removed, a C-1 and a partial C-2 laminectomy was then performed. The right cerebellar tonsil was dissected free of the dorsal medulla and a gush of cerebrospinal fluid gave good decompression of the posterior fossa content.
61343 (Craniectomy performed for Chiari malformation is assigned code 61343, located under the Nervous System, Skull, Meninges, and Brain, Craniectomy/Craniotomy Procedures.)
Craniofacial approach to the anterior cranial fossa, extradural with resection of lesion of anterior cranial fossa, extradural
61600, 61580-51 (Assign code 61600 for resection lesion, anterior fossa located in the Definitive Skull Base, Anterior Fossa Procedure section. Also assign 61580-51 for craniofacial approach.)
Laminectomy performed on L1-L2, L2-L3
63005 (Lumbar laminectomy performed on one to two levels is assigned 63005.)
Ectropion of the right lower eyelid. Repair with tarsal wedge excision is performed for correction. Attention was then directed to the left eye. The patient also had an ectropion of the left lower lid that was repaired by suture repair.
67916-E4, 67914-E2 (Two ectropions were repaired, one of the right lower eyelid with tarsal wedge to repair, code 67916-E4, and one with suture of the lower left, code 67914-E2.)
Possible basal cell carcinoma of right upper lid. Area was marked in a wedge shape, and a wedge resection was performed with Mescott scissors. Three sutures were used to close the tarsus, then two silk sutures were passed through the anterior/posterior lid margins to oppose the lid margins together. Vicryl sutures were used to close the orbicularis oculi layer superior to the tarsus.
67961-E3 (Performed on the eyelid and tarsal; therefore, codes from the ocular/eye section are utilized. Modifier -E3 is utilized to specify the right upper lid.)
A patient presents for probing of nasolacrimal duct for possible obstruction.
68810-LT (Probing of the left nasolacrimal duct, assigned 68810, is found in the Eye Section, Lacrimal System, Probing section.) WRONG--question doesn't say left)
3 V of right knee. There is a 0.3 cm calcification over the central knee portion. Could be an avulsion fracture. Recommend AP view be repeated with notch view.
73562-RT (Code 73562 for 3 view knee with modifier -RT appended to indicate right)
When impacted cerumen is removed by surgical instrumentation, what code(s) would be appropriate?
69210 (Impacted cerumen with the use of surgical instrument is assigned code 69210.)
Removal of impacted cerumen by surgical instrumentation
69210 (When impacted cerumen is removed by surgical instrumentation, code 69210 should be assigned from the Ear, Removal External Ear section.)
A patient with severe mental retardation and cerumen impactions in both ears was unable to be cleared in the office. Using surgical instrumentation, the canal was cleared of impacted cerumen bilaterally.
69210-50 (Assign code 69210-50 for bilateral removal of impacted cerumen. Cerumen was cleared with surgical instrumentation and therefore was assigned code 69210.)
Bilateral microscopic ear examination and removal of bilateral cerumen impactions. Utilizing the ear speculum and surgical forceps, the external canal was cleared of impacted cerumen by surgical forceps.
69210-50 (When surgical instrumentation is utilized to remove impacted cerumen, code 69210 is assigned. Since this was performed bilaterally, assign 69210-50.)
Placement of ventilation tube, left ear, with local anesthesia
69433-LT (Placement of a ventilation tube, also known as a tympanostomy (create an artificial opening), is assigned 69433-LT when performed with local anesthesia only.)
Radial incision was made in the posterior quadrant of the left and right tympanic membrane. A large amount of mucoid effusion was suctioned and then a ventilating tube was placed in both ears under general anesthesia.
69436-50 (Assign 69436-50 as myringotomy with tube placement, also referred to as a tympanostomy, is performed bilaterally.)
After general anesthesia, the right ear was cleansed of excess cerumen with cerumen curette and alligator forceps. Ear canal was cleared and the tympanic membrane was retracted with thin effusion in the middle ear space. Myringotomy knife was used to create a radial incision in the anterior inferior quadrant, followed by evacuation of the middle ear fluid with suction. An Armstrong Beveled Grommet ventilation tube was carefully inserted through the myringotomy incision. Similar procedure was performed on the left ear.
69436-50 (Tympanostomy with tube placement was performed bilaterally; therefore, 69436 would be reported along with modifier -50. The removal of cerumen was not the definitive procedure but necessary to visualize the surgical site and, therefore, is not separately reportable.)
Modified radical mastoidectomy, left
69505-LT (When a mastoidectomy is performed that is specified as modified radical, assign code 69505-LT.)
A patient has been definitively diagnosed with Meniere's disease. The patient presents for a transcanal chemical labyrinthotomy to the right ear. Dr. Miller visualizes the tympanic membrane with an operating microscope, cleans the ear canal, and makes a small incision into the tympanic membrane. Gentamicin is delivered into the right ear. The perfusion is repeated to achieve the maximum result.
69801, 69990 (Assign code 69801 for the labyrinthotomy as well as 69990 as an operating microscope was utilized to visualize and perform the procedure.)
Tumor removal from left temporal bone
69970 (Assign code 69970 for removal of tumor from the temporal bone.)
When anesthesia is calculated in 15 minute increments, 11:00 to 12:45 would be calculated as how many units?
7 (A total of 7 time units would be assigned, 11:00 to 12:00 = 4 units, 12:00 to 12:45 - 45 minutes = 3 units, total 7 units)
CT brain is performed with and without contrast.
70470 (CT brain with and without contrast is assigned 70470 from Diagnostic Radiology, head/brain section.)
MRI brain without contrast
70551 (Assign 70551 for MRI brain w/out contrast from Diagnostic Radiology, head/brain section.)
Portable chest, one view. Cardiac silhouette is prominent, unchanged since prior exams. No superimposed pulmonary edema or infiltrate is evident.
71045 (Portable chest X-ray is considered one view, and, therefore, 71045 should be assigned from Diagnostic Radiology section, Chest.)
MRI chest
71550 (MRI chest assigned 71550 without contrast)
X-ray of thoracic spine 2 V
72070 (Assign 72070 for thoracic spine, 2 view X-ray found in Diagnostic Radiology, spine section, thoracic.)
2 V left shoulder. Moderate degenerative changes of the left AC joint noted.
73030-LT (Assign 73030-LT for 2 view left shoulder from Diagnostic Radiology, upper extremity, shoulder section.)
MRI, elbow joint, right, with and without contrast, professional component only
73223-26 (Would assign 73223 for MRI elbow with and without contrast from Diagnostic Radiology, MRI upper extremity joint. Also would append modifier -26 for professional component only.)
X-ray, hip with pelvis, 2 V, left
73502-LT (Diagnostic X-ray, hip, 2 view with pelvis, assigned 73502-LT)
X-ray, knee, 3 V, right
73562-RT (Diagnostic X-ray, knee, 3 views, assigned 73562-RT)
Supervision and interpretation aortography thoracic
75600 (Supervision and interpretation for thoracic aortography is assigned 75600.)
Breast ultrasound, right
76641-RT (Ultrasound, breast, assigned 76641-RT)
Renal ultrasound. Small area of echogenicity with shadowing in the midleft kidney that could represent a small calculus. Exam of right kidney is negative. Urinary bladder is not visualized.
76775 (Renal ultrasound assigned 76775. Considered limited abdominal retroperitoneal as not all organs ordered or documented.)
Diagnostic mammogram with CAD, right
77065-RT (Assign 77065-RT for unilateral diagnostic mammogram.)
Left and right mammogram for benign fibrocystic breast disease
77066 (Diagnostic mammogram code 77066 is already bilateral, therefore, no need for modifier -50.)
Bilateral screening mammogram
77067 (Screening mammogram, bilateral, no need to assign modifier -50 as code is bilateral.)
A 69-year-old Medicare patient presents for annual mammogram, indicating she is leaving for an extended trip and would like her screening mammogram performed before the one-calendar-year period. Assign the appropriate CPT code(s) for this service.
77067-GA (It would be appropriate to assign the GA modifier to the claim after obtaining a signed ABN from the patient as the service will be denied. The mammogram will be considered screening 77067-GA since there is no history of a medical problem.)
Radiation treatment, two treatment areas, 20 MeV
77407 (Code 77407 assigned for radiation treatment to two treatment areas, 20 MeV from radiation oncology section, radiation treatment category)
Proton treatment delivery, simple without compensation
77520 (Proton treatment delivery assigned code 77520 for radiation oncology section. Proton delivery simple without compensation.)
Comprehensive metabolic panel, CBC, automated differential WBC, and TSH (thyroid stimulating hormone)
80053, 85025, 84443 (A comprehensive metabolic profile is performed and coded to 80053. In addition, CBC with automated differential WBC is performed and coded to 85025 as well as TSH (thyroid stimulating hormone) codable to 84443.)
Acute hepatitis panel
80074 (Code 80074 is assigned for acute hepatitis panel from the Disease-Oriented Panel section of Pathology.)
Drug screening, three drug classes, ELISA method
80307 (Code 80307 is assigned for drug screening by ELISA method regardless of the number of drug classes tested.)
Drug confirmation, benzodiazepines, 10 analytes
80346 (Drug confirmation for 10 analytes of benzodiazepines is assigned 80346 that is for 1-12 analytes.)
BRCA2 gene analysis, full sequence analysis
81216 (Code 81216 is assigned for an analysis of the BRCA2 gene when full sequence analysis is performed per CPT Pathology guidelines.)
Arsenic chemistry analysis
82175 (Chemical analysis of arsenic is assigned code 82175 from the Chemistry section of Pathology.)
Carbon dioxide, chloride, potassium, sodium
82374, 82435, 84132, 84295 (As carbon dioxide, chloride, potassium, and sodium are not a disease-oriented panel, each service must be assigned individually as 82374, 82435, 84132, and 84295, respectively.)
Reticulocyte count, automated
85045
PTT (thromboplastin time, partial), whole blood
85730 (Code 85730 is assigned for partial thromboplastin time or PTT.)
Antinuclear antibody titer
86039
Antibody, chlamydia
86631 (Chlamydia antibody is assigned 86631.)
Antibody CMV (cytomegalovirus)
86644 (CMV (cytomegalovirus) antibody is assigned code 86644.)
Antibody for Epstein-Barr virus, nuclear antigen
86664 (Antibodies for Epstein-Barr by nuclear antigen is assigned code 86664.)
Blood typing ABO
86900
Infectious agent antigen detection, herpes simplex virus type 2, by immunofluorescent technique
87273 (Herpes simplex virus type 2 antigen detection by immunofluorescent technique is assigned 87273.)
Infectious agent antigen detection by immunoassay, Helicobacter pylori, ELISA, stool
87338 (Helicobacter pylori, stool, antigen detection by immunoassay is assigned 87338.)
Left heart catheterization for PTCA of right coronary and intracoronary stent
92928-RC (Heart catheterization performed for purpose of angioplasty and stent to right coronary. Only successful intervention is assigned; therefore, only stent, 92928-RC is assigned.) WRONG
Surgical pathology, carpal tunnel
88304 (Surgical pathology for carpal tunnel is assigned 88304 per surgical pathology listing in CPT.)
Surgical pathology, oophorectomy, non-neoplastic
88305 (Surgical pathology for oophorectomy is assigned 88305 per surgical pathology examples in CPT.)
Surgical pathology submitted to the pathologist is as follows: three surgical specimens taken from a liver biopsy.
88307 (Code 88307 only is assigned for the liver biopsy, as each specimen was not individually identified.) WRONG
Surgical pathology, needle biopsy, liver
88307 (Surgical pathology for liver needle biopsy is assigned 88307 per surgical pathology listing in CPT.)
A patient with a large colonic tumor, identified by CT, presented for surgical resection. During the surgery, the physician also excised a soft tissue mass from the abdomen. The pathologist performed a gross and microscopic examination of the following specimens: (a) colon, terminal ileum, submitted in two cassettes and (b) mass, soft tissue, submitted in one cassette.
88309, 88307 (Two specimens were separately identified, one malignant tumor of the colon, assigned 88309, and one soft tissue mass of the abdomen, assigned code 88307.) WRONG
A 17-year-old presents for his initial visit with complaints of left knee pain after playing football. An expanded problem-focused history and exam were performed with straightforward MDM.
99202 (All three elements are met for 99202.)
Psychotherapy, 45 minutes, outpatient office
90834 (Code 90834 should be assigned for 45 minutes of psychotherapy.)
Psychotherapy, 45 minutes, office
90834 (Forty-five minutes psychotherapy assigned 90834.)
Gastric motility study
91020 (Assign 91020 for gastric motility study from GI Procedures.)
Comprehensive ophthalmological exam, new patient
92004 (Code 92004 is assigned for comprehensive ophthalmological exam performed for a new patient.)
Prescription and fitting of contact lenses
92310 (For the prescription and fitting of contact lenses, assign 92310.)
Speech audiometry threshold
92555 (Assign 92555 for speech audiometry threshold from the Audiologic Function Testing section.)
A patient presents with diagnosis of allergic sinusitis from his or her PCP for allergy testing. Twelve percutaneous scratch allergy tests with allergenic extracts are performed.
95004 X12 (Percutaneous scratch testing with allergenic extracts is assigned per test; therefore, 95004 should be assigned with 12 units.)
When patient brings his or her own allergy medication and the only service provided is the injection of the allergan.
95115 (Code 95115 is assigned for allergy immunotherapy when administration performed only.) WRONG
Allergy immunotherapy with no provision of extract as well as a problem-focused history, exam, and low MDM
95115, 99212-25 (E/M 99212-25 would be assigned to indicate "significantly, separately identifiable" E/M has been performed. Also assign 95115 for allergy immunotherapy with no provision of extract.)
Allergy immunotherapy, administration of antigens requiring three injections
95117 (Code 95117 is assigned for administration of more than two injections.) WRONG
Nerve conduction studies, two studies
95907 (Nerve conduction studies of one to two studies are assigned 95907.)
A patient presents with cough/fever and is diagnosed with pneumonia. IV normal saline 2:00-2:45, IV infusion Rocephin 3:00-3:45.
96365, 96361 (For physician coding purposes, the primary service is the service that prompted the encounter. Since the patient presented for cough/fever and was diagnosed as pneumonia, the IV Rocephin antibiotic would be the primary service; therefore, the IV infusion would be primary, with the IV hydration secondary.)
IV infusion for a total of 3 hours
96365, 96366 x2 (Three hours of IV infusion should be assigned 96365 for the first hour and 96366 x2 for two additional hours.)
A patient with Paget's disease of the bone receives an intravenous infusion of Aredia that begins at 12:29 PM and ends at 4:11 PM. Code this drug administration service.
96365, 96366 x3 (Assigned 96365 for first hour, and 96366 for 3 additional hours of IV infusion.) WRONG
A patient is diagnosed with bronchial asthma following evaluation. The patient is administered 20 mg of Depo-Medrol (methylprednisolone acetate) intramuscularly.
96372, J1020 (Assign 96372 for the IM injection and J1020 for the 20 mg of Depo-Medrol.)
IV push, antineoplasm (chemotherapy) drug
96409 (Assign code 96409 for intravenous push, injection from the Chemotherapy Administration section.)
Chemotherapy infusion therapy, 9:00-10:45
96413, 96415 (Code 96413 is assigned for first hour; code 96415 for additional hour (minimum 30 minutes needed).)
Dermatological ultraviolet light treatment
96900 (UV light treatment is assigned 96900.)
Physical therapy evaluation, low complexity
97161 (Assign 97161 for physical therapy evaluation when low complexity.)
Debridement of devitalized epidermis, open wound, selective, 20 sq cm
97597 (20 sq cm of selective active wound therapy is assigned code 97597.)
Prosthetic training, 60 minutes
97761 x4 (Code 97761 should be assigned a total of four units as the code is per 15-minute increments.)
The patient is a 58-year-old white male, 1 month status post pneumonectomy. He had a post pneumonectomy empyema treated with a tunneled cuffed pleural catheter, which has been draining the cavity for 1 month with clear drainage. He has had no evidence of a block or pleural fistula. Therefore, the patient returned to the physician and had the removal of the catheter performed.
99024 (If the same physician placed the pleural catheter, it would be considered in the global procedure, and therefore, only the code 99024 for postoperative visit would be appropriate.)
A patient had an open acromioplasty of her shoulder approximately 9 days ago. The incision is healing well and sutures were removed today. She is going to start physical therapy and was given a prescription for ibuprofen 800 mg tid to be taken as needed. Today's visit consisted of an expanded problem-focused history, expanded problem-focused exam, and low MDM.
99024 (Regular postoperative evaluation. No E/M service is assigned, only 99024 to track visit.)
Qualifying circumstances code assigned for patients under 1 or over 70
99100
New office visit for patient with lupus with kidney disease, edema with cardiac problems. History was problem focused for this visit. Exam encompasses skin, CV, renal, neurovascular systems. Significant PMH and PHF of lupus and cardiac disease. No tests are ordered or interpreted, management/treatment options are moderate as well as risk.
99201 (As this is a new office visit, all three elements (history, exam, and medical decision making) are required. As the history was problem focused only, the level of service would be limited to 99201.) WRONG
Patient presents for initial evaluation complaining of severe pruritic rash on her left and right antecubital fossa x 4 days. Reports rash began on her arms, erupted and spread to her forearms and lower extremities. Also has multiple lesions across her chest area. Denies any new lotions, soaps, foods, pets, or clothes. No previous history of dermatitis. Denies fever, chills, other than rash, all other systems are unremarkable. Exam: diffuse vesicular lesions across upper torso, forearms, and thighs. Erythematous area of lesions in her right and left popliteal fossa. Vesicular lesions, etiology unknown. If no improvement in next 24-48 hours should consider dermatological consult.
99201 (The exam was problem focused as only one body system (integumentary/skin) was examined despite the fact the skin was located on multiple areas of the body.)
Patient admitted to observation care at 11:00 PM for chest pain. Detailed history, comprehensive exam, and low MDM are performed. Patient is discharged at 11:00 AM the following day.
99218, 99217 (As two different dates of service, two codes would be assigned. Observation limited by low MDM to 99218 and observation discharge 99217.)
Patient admitted to ICU for observation and monitoring purposes on 01/01.
99221 (Documentation does not support critical care guidelines as listed in CPT, and would be considered an initial hospital visit. Therefore, code 99221 would be the only appropriate answer.)
An established patient is seen in the hospital for subsequent visit. Detailed history and exam are performed and medical decision-making is of high complexity.
99233 (All three elements qualify for 99233.)
ER visit, detailed history, detailed exam, low MDM
99282 (Three of three elements needed, low MDM makes visit 99282.)
ED visit, expanded problem-focused history, detailed exam, low MDM
99282 (Visit limited by expanded problem-focused Hx to 99282.)
Emergency department visit, expanded problem-focused Hx, detailed exam, high MDM
99283 (Hx - Expanded Problem Focused - 99283 Exam - Detailed - 99284 Medical Decision-Making - High - 99285 Emergency Department visits require all three elements to be met; therefore, would qualify for 99283 only.) WRONG
Emergency department visit for fractured ankle. Detailed history, expanded problem-focused exam, moderate MDM.
99283 (Visit limited by expanded problem-focused exam to level 99283. All three elements, Hx, exam, and MDM must be met for an Emergency Room Visit.)
Dr. Smith spent 1 hour and 30 minutes delivering critical care to a 48-year-old patient. Over the course of this time, Dr. Smith provided interpretation of cardiac output measurements and withdrawal of arterial blood. Dr. Smith did not attend other patients during this time.
99291, 99292 x 1 (According to the time grid in the critical care section, critical care from 75 to 104 minutes qualifies for 99291 and 99292 x 1.)
Established patient presents for 3-year-old annual exam. Comprehensive history and exam and moderate MDM were performed based on CPT guidelines. Appropriate anticipatory guidance was provided during the encounter. Assign the appropriate E/M code(s) and modifier(s).
99392 (Since the patient is established, a preventive medicine exam for a 3-year-old would be appropriate, code 99392.)
Annual preventive medicine exam for a 38-year-old established male patient.
99395 (Preventive medicine visit for established 38-year-old is 99395.)
Which code(s) would be utilized for a critically ill 20-day-old patient initial care?
99468-99469 (Per CPT 20 day is still considered infant, therefore, neonatal critical care codes 99468-99469 would be utilized for initial care.)
Additional authoritative coding guidelines for procedures may be found in what publication?
AMA CPT Assistant (CPT Assistant provides additional information and clarification on the appropriate assignment of CPT codes.) WRONG
Which of the following situations could be considered upcoding? A podiatrist performs simple nail clipping services but bills for foot surgery. A provider bills for services as though a physician rendered them, even though there was no physician involvement. A provider bills Medicare for treating a patient with a more serious form of pneumonia than the patient has. All of those listed.
All of those listed.
A psychoneurotic disorder characterized by prolonged refusal to eat resulting in extreme weight loss is known as
Anorexia nervosa (Anorexia Nervosa is characterized by excessive weight loss due to prolonged periods of refusing to eat. Hypochondria is a psychiatric disorder where one has abnormal anxiety about one's health; bulimia is typically referred to as the "binge and purge" disorder; and schizophrenia is a breakdown in the relation to thought, emotion, and behavior.)
Neurosurgeon and otolaryngologist both participate in performing a cochlear implant on a 6-year-old female. Neurosurgeon codes and submits 69930-RT and is paid 100% of the fee schedule allowance. Otolaryngologist submits code 69930-RT and is denied payment for "services paid to another provider." What was incorrect in regard to the coding for these services?
Both providers must append modifier -62 for both providers to be paid appropriately. (Both physicians (two surgeons) performed a portion of the same service and, therefore, share in the code/reimbursement for the service. A modifier -62 should be assigned to the claims for both physicians to be reimbursed appropriately.)
Breast prosthesis (implanted) following mastectomy
C1789 (Assign C1789. Other codes listed are for implantation of other devices.)
Specific E & M guidelines utilized by Medicare/CMS may be found where?
CMS website (The CMS website provides specific guidelines regarding E/M service for Medicare.)
Anesthesia coding and billing always require the following elements:
CPT code, physical status modifier, and time units (A CPT code, physical status modifier, and time will always be reported.) WRONG
Which of the following is NOT a part of the computer planning service for intensity-modulated radiation therapy (IMRT)? Verification of treatment setup and interpretation of verification methodology Creating highly conformal radiation dose distribution Inverse treatment planning Verification of positional accuracy
Creating highly conformal radiation dose distribution WRONG
A 65-year-old patient was presented for removal of peripheral central venous catheter. Catheter was removed with no problems encountered.
E/M only (Only tunneled central venous catheter may be coded for removal; therefore, only an E/M would be appropriate.)
Pair of forearm crutches with tips and handgrips was provided during the patient's visit.
E0110 (E0110 is the only code for forearm crutches.)
Hospital bed, with rails and mattress
E0250 (Bed is with rails and mattress; therefore, E0250 is appropriate.)
Encounter with double vision (diplopia) of a 10-year-old due to ingestion of the parents' Prozac
H53.2, T43.221A (If the intent of the poisoning is unknown or unspecified, code the intent as accidental intent. Sequence second code(s) for all manifestations after the poisoning.)
A 78-year-old female with decreased visual acuity. Diagnosis of macular degeneration made approximately 1 year ago. Patient presents for next macular degeneration treatment. Gave IV 7 cc selenium with additional 5 cc bacteriostatic water. Will repeat again in 1 week.
IV Only (Patient scheduled for procedure, therefore, procedure only assigned.)
Which of the following drugs, biologicals, or substances would not be reported with chemotherapy administration procedure codes?
IVP of an antibiotic (An antibiotic is not considered a chemotherapy drug; therefore, code would not be assigned from chemotherapy administration section.)
URI determined to be the cause of pharyngitis
J06.9 (Not necessary to assign code for pharyngitis as inflammation of throat would be sign/symptom of the URI) WRONG
Injection, ceftazidime, 750 mg
J0713 X 2 units (J0713 per 500 mg. Since total of 750 mg administered, J0713 x 2 would be appropriate.) WRONG
Pneumonia, probably streptococcal
J18.9 ("Probably" cannot be coded, therefore, code only pneumonia.)
Patient presents with left lower knee pain. An arthrocentesis is performed with the injection of 10 mg of Kenalog. Code the Kenalog (triamcinolone acetonide).
J3301 (J3301 covers up to and including 10 mg of Kenalog; therefore, only one unit is reported of J3301.)
Injection, diazepam 7.5 mg
J3360 X 2 units (Code J3360 is per 5 mg; therefore, two units must be assigned. Correct code is J3360 x 2.)
Exacerbation of COPD
J44.1 (Code as exacerbation, disease, pulmonary, obstructive, chronic.)
Food aspiration pneumonia
J69.0 (Diagnosis is specified as pneumonia "due to" food aspiration; therefore, pneumonia would not be appropriate, and pneumonia due to food would be assigned.)
Repair of nonoxygen, durable medical equipment, 30 minutes
K0739 X 2 units (Code K0739 is for 15 minutes DME repair. Therefore, K0739 x 2 would be appropriate.)
Abdominal pain, gastroenteritis, probably viral
K52.9 ("Probably" cannot be coded; therefore, code gastroenteritis only. Abdominal pain is sign/symptom of gastroenteritis and does not need to be coded.) WRONG
Scope advanced into cecum. There were multiple diverticula seen in the sigmoid and descending colon, some were filled showing diverticulitis.
K57.92 (Diverticula has developed into diverticulitis, therefore, no need to code the diverticula.) WRONG
Patient presents with concerns of rectal bleeding and diarrhea. Patient has a past history of colon cancer diagnosed 12 years ago, treated with surgery and has remained cancer free. Patient will undergo colonoscopy. Until that time, patient will refrain from anti-diarrhea medications and use Preparation H for what are believed to be hemorrhoids.
K62.5, R19.7, Z85.038 (Hemorrhoids cannot be coded as "believed to be." Code signs/symptoms for presenting problem: rectal bleeding and diarrhea. Also code history of colon cancer as increased complexity of medical decision making.)
Patient with history of colonic polyps presents for colonoscopy with symptoms of bloody stools and abdominal pain.
K92.1, R10.9, Z86.010 (Primary code would be presenting symptoms: bloody stools and abdominal pain. Also code for history of colonic polyps as contributed to reason for colonoscopy.)
Cervical collar, two piece prefab, semirigid foam
L0172 (L0172 is the only code that encompasses all the elements listed in the descriptor, namely, two piece, prefab, and semirigid foam.)
Infected scalp laceration from fall 3 weeks ago
L08.9, S01.01XA, W19.XXXA (Active treatment of the laceration necessitates the seventh character "A" even though the initial injury was three weeks prior.)
Partial hand prosthesis, thumb remaining
L6000 (L6000 only code for partial hand, thumb remaining)
Breast prosthesis, mastectomy sleeve
L8010 (L8010 only code for breast mastectomy sleeve)
X-ray of left knee for pain following fall. History of TKA 2 years ago. AP and lateral views of left knee demonstrate left knee replacement in satisfactory position. No fracture, subluxation seen.
M25.562, W19.XXXA, Z96.652 (No definitive diagnosis on x-ray; therefore, utilize left knee pain. Total knee replacement is significant for additional complexity for encounter and is evaluated. Fall not coded as external cause codes assigned for current event only.)
Right kidney mass
N28.89 (No code for mass, kidney. Therefore, code to other conditions/diseases of kidney/ureter.) WRONG
Cervical epidural steroid injection and epidurogram were performed under fluoroscopic guidance. Would the epidurogram be codeable in conjunction with the cervical epidural steroid injection?
No (No, the epidurogram is performed for identification and verification of the site only and, therefore, is not separately reportable.) WRONG
Code 29880 was performed in the medial compartment as well as 29875. What modifier(s), if any, would be appropriate for 29875?
None (No modifier would be appropriate because 29875 is designated as a "separate procedure", and therefore is not codable with 29880.) WRONG
The degenerative motor system disorder that is the slow progression of the deterioration of neurons in the substantia nigra area of the brain is known as
Parkinson's disease (Parkinson's disease is a degenerative motor disorder that deteriorates the neurons.)
A cardiologist is called to the ED to perform a consultation. The service was coded as follows: 01/01/XX with Place of Service Inpatient Hospital, and CPT Code 99251. Why will the claim be denied?
Place of service and CPT code are incorrect. (Emergency room services are considered outpatient, and, therefore, the place of service 21 is incorrect as well as the CPT code 99251 that is for inpatient consultations. Outpatient consultation codes 99241-99245 should be reported.)
In order for physician supervision requirements to be met for nonprovider staff, what conditions must be met?
Provider must be in the physician suite. (In the event nonprovider staff, such as an RN provides service, the provider must be in the office suite at the time the services are performed. It is not acceptable for the provider/physician to be only in the same building or available for contact.) WRONG
A 35-year-old patient is diagnosed with fractured distal radius following radiological studies. A short-arm fiberglass cast is constructed and applied.
Q4010 (Q4010 is assigned to short-arm fiberglass cast for an adult.)
Portable chest x-ray for history of shortness of breath, R/O cardiomegaly. Cardiac silhouette is prominent, unchanged since prior exams. No superimposed pulmonary edema or infiltrate is evident.
R06.02 (No definitive diagnosis on x-ray; therefore, utilize shortness of breath. Cardiomegaly stated as "rule out"; therefore, cannot be assigned per ICD10 guidelines.)
Encounter due to family history heart disease. Patient experiencing chest pain and chest tightness.
R07.9, R07.89, Z82.49 (Chief reason for encounter are symptoms of chest pain and chest tightness. Family history of heart disease as to complexity of visit and should be coded last.)
Headache NOS
R51 (No additional information documentation; therefore, assigned code for "Headache" only, R51.)
Abnormal chest x-ray
R91.8 (Located under Abnormal Radiological Findings, Chest) WRONG
Fracture left distal radius following fall from bus
S52.502A (Code needs to encompass "distal" (Lower end) radius as well as seventh digit for treatment period and anatomical location "left.")
Fracture of the right tibial shaft as a result of a fall
S82.201A (Fracture is considered traumatic rather than pathological. Fracture all needs seventh character to denote treatment period.)
Patient presents for knee arthroscopy after several months of left knee pain. Arthroscope is introduced and the medial joint has some synovium and a torn complex medial meniscus that was repaired.
S83.232A (Primary reason for encounter is knee arthroscopy for findings of complex torn medial meniscus.)
When CRNAs (certified registered nurse anesthetists) provide anesthesia services under the direction of the anesthesiologist, how will these services be coded/billed?
Same anesthesia code with modifier to designate CRNA (CRNAs utilize the same anesthesia CPT codes with a modifier to designate CRNA.)
What is one factor of the federal guidelines for compliance programs?
The organization has communicated standards and procedures to employees and agents. (The organization must communicate the standards and procedures to their employees and any agents of the practice.)
Level II surgical pathology codes are utilized for specimens that are not removed for suspected malignant, but other reasons.
True
When locating an open wound in ICD10 in the alphabetic section, the coder would look under
wound, open (Lacerations and open wounds are now distinguished by separate codes in ICD10. Coder should look under "wound, open.")
Child presents to physician due to exposure to chickenpox.
Z20.820 (Code Z20.820 can be found under "Exposure to.")
A sudden loss of kidney function is called
acute renal failure (Acute kidney failure is a sudden or abrupt loss of kidney function. Chronic renal failure refers to a longstanding renal failure. Cystitis is the inflammation of the kidney and not failure.)
The surgical removal of a gland or gland part is called
adenectomy (All choices involve the removal/excision of a body part, however, "adeno" refers to gland. Therefore, the correct choice is adenectomy, which is the removal of a gland or part of a gland.)
Gastro pertains to
the stomach (The term "gastro" refers specifically to the gastric or stomach, which is part of the digestive/GI system.)
Modifier -FA denotes
thumb, left hand (The modifier "FA" denotes the thumb of the left hand, per CPT. Toes are designated with "T" modifiers and the thumb of the right hand is designated as "F5.")
Onco is defined as a(n)
tumor (The prefix "onco" refers to a tumor, not necessarily cancer because the tumor could be benign.)
When there is no CPT code that specifically describes the services performed, what should be assigned?
unlisted procedure code (Unlisted CPT codes should be assigned when there is no CPT code that adequately describes the services performed. HCPCS codes are assigned for non-surgical and procedural services not listed in CPT. While the claim may need to be submitted with an accompanying procedure report, the question asks what should be assigned not attached.)
When a vasectomy is performed, these ducts are cut and/or fulgurated to prevent sperm from reaching the ovum.
vas deferens (The vas deferens are "interrupted" to prevent sperm from reaching the ovum and completing the fertilization process.)
An idiopathic disorder refers to a disease/disorder that is
without known cause (Idiopathic means "without cause." An idiopathic disease is not caused by external forces, psychological factors, or self-inflicted. The disease at the time of diagnosis has no known cause.)